You are on page 1of 43

Incremental Analysis

MODULE 6 24. One of the behavioral problems with relevant cost analysis is the overemphasis on short-term
goals, which can lead to neglect of:
INCREMENTAL ANALYSIS A. sales promotion C. quarterly net income results
B. expense control D. long-term strategic goals
Basic concepts
Steps in decision making process Incremental analysis
5. What is the first step in the decision making process? 25. Incremental analysis is the process of identifying the financial data that:
A. Specify the criteria by which the decision is to be made. A. do not change under alternative courses of action
B. Consider the strategic issues regarding the decision context. B. are mixed under alternative courses of action
C. Perform an analysis in which the relevant information is developed and analyzed. C. change under alternative courses of action
D. Compare the alternatives. D. no correct answer is given

7. A major accounting contribution to the managerial decision-making process in evaluating 48. Incremental analysis is most useful
possible courses of action is to A. in evaluating the master budget.
A. assign responsibility for the decision. B. in choosing between the net present value method and the internal rate of return method.
B. provide relevant revenue and cost data about each course of action. C. in developing relevant information for management decisions.
C. determine the amount of money that should be spent on a project. D. as a replacement technique for variance analysis.
D. decide which actions that the management should consider.
Relevant information
8. An analysis of relevant costs and relevant revenues 2. Predicted future cost and revenue data that will differ among alternative courses of action are
A. Will enable the decision maker to assess a decisions impact on profit known as
B. Is useful in assessing a variety of alternative decisions A. relevant information C. marginal costs
C. Provides sufficient and complete evidence with which to make a decision B. direct information D. incremental costs
D. Answers a. and b. are correct
4. Which of the following is described as data that are pertinent to a decision?
Pitfalls in decision making A. qualitative characteristics C. timely information
1. When discussing the pitfalls to be avoided in decision-making, four reminders usually emerge. B. accurate information D. relevant information
Which is NOT one of those reminders?
A. Ignore sunk costs. 6. Which of the following best describes relevant information?
B. Beware of allocated fixed costs; identify the avoidable costs. A. Focused on the past and differs between the alternatives under consideration.
C. Pay special attention to identifying and including opportunity costs. B. Focused on the past and not related to the decision under consideration.
D. Do not overlook the time value of money in short-run decisions. C. Focused on the future and differs between the alternatives under consideration.
D. Focused on the future and not related to the decision under consideration.
19. Which one of the following is not a common mistake in a decision-making process?
A. Considering sunk costs as relevant. Application of incremental analysis
B. Considering opportunity cost, an imputed cost, being relevant. 3. Incremental analysis would not be appropriate for
C. Considering fixed costs as avoidable fixed costs. A. a make or buy decision.
D. Unitizing fixed costs. B. an allocation of limited resource decision.
C. elimination of an unprofitable segment.

11
Incremental Analysis

D. analysis of manufacturing variances. Relevant costs


16. Relevant costs are
Irrelevant costs A. all fixed and variable costs
Sunk costs B. all costs that would be incurred within the relevant range of production
9. The kind of cost that can be ignored in a short-term decision making is a(an) C. past costs that are expected to be different in the future
A. differential cost C. sunk cost D. anticipated future costs that will differ among various alternatives
B. incremental cost D. joint cost
14. The Health Care Division of Piedmont Insurance employs three claims processors capable of
30. Sunk costs are processing 5,000 claims each. The division currently processes 12,000 claims. The manager
A. Costs that increase due to a higher volume of activity or the performance of an additional has recently been approached by two sister divisions. Auto Division would like the Health
activity Care Division to process approximately 2,000 claims. Property Division would like the Health
B. Costs that a company must incur to perform an activity at a given level, but will not be Care Division to process approximately 5,000 claims. The Health Care Division would be
incurred if a company reduces or discontinues the activity compensated by Auto Division or Property Division for processing these claims. Assume that
C. The profits that a company forgoes by following a particular course of action these are mutually exclusive alternatives. Claims processor salary cost is relevant for
D. Costs that were incurred prior to making a decision A. Auto Division alternative only
B. Property Division alternative only
33. A sunk cost is: C. both Auto Division and Property Division alternatives
A. a cost incurred in the past and not relevant to any future course of action. D. neither Auto Division nor Property Division alternatives
B. an opportunity cost.
C. useful in analysis of alternative courses of action. Differential costs
D. relevant to current decision making. 31. The difference in cost between or among various alternative courses of action appropriately
describes a(an):
13. Which of the following is least likely to be a relevant item in deciding whether to replace an old A. differential cost C. constraint
machine? B. ad hoc discount D. scarce resource
A. acquisition cost of the old machine
B. outlay to be made for the new machine Opportunity cost
C. annual savings to be enjoyed on the new machine 10. An important concept in decision making is described as the contribution to income that is
D. life of the new machine forgone by not using a limited resource in its best alternative use. This concept is called
A. Marginal cost C. Incremental cost
Unit costs B. Cost outlay D. Opportunity cost
22. Unit costs can mislead decision makers. Which of the following situations dealing with unit
costs are not expected to result in a faulty analysis? 11. An opportunity cost is
A. Unit costs used in make-or-buy decisions might include costs such as avoidable fixed A. the difference in total costs that results from selecting one alternative instead of another
costs. B. the profit forgone by selecting one alternative instead of another
B. Variable unit cost directly varies with the changes in production units. C. a cost that may be saved by not adopting an alternative
C. Total fixed costs increase as more units are produced within the relevant range. D. a cost that may be shifted to the future with little or no effect on current operations
D. Contribution margin on products that can be manufactured in using the freed capacity is
irrelevant in the decision. 12. The best characterization of an opportunity cost is that it is
A. relevant to decision making but is not usually reflected in accounting records

12
Incremental Analysis

B. not relevant to decision making and is not usually reflected in accounting records B. II D. II and III
C. relevant to decision making and is usually reflected in accounting records
D. not relevant to decision making and is usually reflected in accounting records 29. Avoidable costs are
A. Costs that increase due to a higher volume of activity or the performance of an additional
18. The potential benefit that may be obtained from following an alternative course of action is activity
called B. Costs that a company must incur to perform an activity at a given level, but will not be
A. opportunity benefit C. relevant cost incurred if a company reduces or discontinues the activity
B. opportunity cost D. sunk cost C. The profits that a company forgoes by following a particular course of action
D. Costs that were incurred prior to making a decision
26. Opportunity cost is the
A. cash outlay required to implement an alternative. Out-of-pocket costs
B. difference in total costs between the alternatives. 23. Which of the following is a cost that requires a future outlay of cash that is relevant for future
C. maximum available contribution to profit that is given up when using limited resources for decision-making?
another purpose. A. Opportunity cost C. Out-of-pocket cost
D. fixed cost avoided when a product, department, or business unit is abandoned. B. Relevant benefits D. Incremental revenue

28. Opportunity costs are Sensitivity analysis


A. Costs that increase due to a higher volume of activity or the performance of an additional 20. Sensitivity analysis is useful in decision making when:
activity A. there is a degree of uncertainty about the relevant data.
B. Costs that a company must incur to perform an activity at a given level, but will not be B. there is an opportunity cost included in the analysis.
incurred if a company reduces or discontinues the activity C. sunk cost is included in the analysis.
C. The profits that a company forgoes by following a particular course of action D. the analysis is subject to a review by the management.
D. Costs that were incurred prior to making a decision
21. To determine the possible outcome in a decision analysis if a key prediction or assumption
27. Using opportunity cost to analyze the income effects of a given alternative is referred to as proves to be wrong, managers will use:
A. engineering analysis C. account analysis A. sensitivity analysis. C. incremental analysis.
B. mixed-cost analysis D. differential analysis B. total analysis. D. regression analysis.

Avoidable Make-or-buy decision


15. A fixed cost is relevant if it is Qualitative Considerations
A. future cost C. avoidable 38. Which of the following elements of the value chain should be considered when deciding
B. sunk D. a product cost whether to make or buy a component needed for production?
A. Marketing C. Manufacturing
17. Which of the following is (are) a true statement(s) about cost behaviors in incremental B. Distribution D. all of these choices
analysis?
I. Fixed costs will not change between alternatives. Make decision
II. Fixed costs may change between alternatives. 34. Manufacturing parts internally by a company causes:
III. Variable costs will always change between alternatives. A. the company to be dependent upon suppliers for timely delivery of parts
A. I C. III B. the quality of the parts to be under the control of the company

13
Incremental Analysis

C. lower parts costs to be assured D. Only conversion costs are relevant.


D. a company's operations to be more efficient than when the parts are purchased from
suppliers Opportunity costs
39. In a make-or-buy decision, which of the following is true?
44. A company should decide to make, rather than buy, a part required for their product, if A. Variable costs are the only relevant costs.
A. The companys production facility is at full capacity B. Allocated fixed costs are relevant.
B. The relevant cost per-unit of making the part exceeds the per-unit relevant costs of C. Alternative uses of space and machinery are relevant.
purchasing the part D. Making the product is the correct decision when there is idle capacity.
C. The supplier of the part can produce a higher-quality part
D. The supplier of the part has questionable reliability 40. The opportunity cost of making a component part in a factory with excess capacity for which
there is no alternative use is
Buy decision A. the total manufacturing cost of the component.
35. In any make or buy decision confronting a company, which of the following factors should be B. the total variable cost of the component.
considered? C. the fixed manufacturing cost of the component.
A. Can the supplier provide a sufficient quantity to meet the company's current and future D. zero.
needs?
B. Do the supplier's items meet product and quality specifications? 46. The cost of not receiving rent from a space because you decide to make the part rather than
C. Is the supplier reliable? buying it from an outside supplier is considered a(an)
D. All of the above should be considered. A. sunk cost C. opportunity cost
B. future cost D. fixed cost
41. Which of the following qualitative factors favors the buy choice in a make or buy decision for a
part? 47. In a make-or-buy decision, an opportunity cost that should be considered is the:
A. maintaining a long-term relationship with suppliers A. income that could be generated from idle production space.
B. quality control is critical B. total costs to produce the item
C. utilization of idle capacity C. variable costs to produce the item
D. part is critical to product D. fixed costs to produce the item

Relevant costs Decision rule


Fixed costs 42. Haribon Company is faced with a make-or-buy decision. Haribon should agree to buy the part
36. Within the context of the make or buy decision, when are fixed costs relevant? from a supplier provided the price is less than Haribons
A. Fixed costs are always relevant A. total costs
B. Fixed costs are never relevant B. variable production costs plus avoidable fixed production costs
C. Fixed costs are relevant when they differ among alternatives C. total manufacturing costs
D. It cannot be determined without closely examining each particular situation D. variable costs

37. In a make or buy decision: 84. A company owns equipment that is used to manufacture important parts for its production
A. Only variable costs are relevant. process. The company plans to sell the equipment for P10,000 and to select one of the
B. Fixed costs that can be avoided in the future are relevant. following alternatives:
C. Fixed costs that will continue regardless of the decision are relevant. (1) acquire new equipment for P80,000

14
Incremental Analysis

(2) purchase the important parts from an outside company at P4 per part. B. Marketing costs D. All of the above
The company should quantitatively analyze the alternatives by comparing the cost of
manufacture the parts Opportunity costss
A. Plus P80,000 to the cost of buying the parts less P10,000. 50. An opportunity cost commonly associated with a special order is
B. to the cost of buying the parts less P10,000. A. the contribution margin on lost sales
C. Less P10,000 to the cost of buying the parts. B. the variable costs of the order
D. To the cost of buying the parts. C. additional fixed cost that is related to the increased output
D. any of the above
Special order decision
Process 53. Operating at or near full capacity will require a firm considering a special order to recognize
49. In making a special order decision, management should: the:
A. compute a reasonable sales price for items not normally produced. A. opportunity cost arising from lost sales
B. consider additional overhead cost. B. value of full employment
C. consider normal and relevant costs. C. time value of money
D. All of the above. D. need for good management

52. Which of the following factors should be considered in deciding whether to accept a special Decision rule
order? 82. Production of a special order will increase gross profit when the additional revenue from the
A. the sales price of the product or service special order is greater than
B. the production capacity of the company A. The nonvariable costs incurred in producing the order.
C. the impact on regular customers B. The direct material and labor costs in producing the order.
D. all of these choices C. The fixed costs incurred in producing the order.
D. The marginal cost of producing the order.
Irrelevant cost
83. In considering a special order that will enable a company to make a use of presently idle 51. If the firm is operating under capacity, the minimum special order price should be high enough
capacity, which of the following costs would be irrelevant. to cover:
A. Materials C. Direct labor A. all variable costs and incremental fixed costs associated with the special order minus
B. Depreciation D. Variable OH foregone contribution margin on regular units not produced.
B. variable and incremental fixed costs associated with the special order and a profit margin.
54. Given the following list of costs, which one should be ignored in a decision to produce C. limited variable costs associated with the special order.
additional units of product for a factory that is operating at less than 100% capacity, and the D. neither variable nor fixed costs associated with the special order.
additional business will not use up the remainder of the plant capacity?
A. Direct material cost per unit C. Fixed selling expenses 57. Green Giant Foods has some excess manufacturing capacity that it can leave idle, use to
B. Direct labor cost per hour D. Variable selling expenses produce its own boxes for frozen foods, or use to process another companys frozen foods. It
will be more profitable for Green Giant to process the competitors frozen foods as long as the
Relevant costs net cost is
Long-run decision A. greater than both the cost to buy the boxes and the cost to leave the plant idle.
58. The sales price of a product, in the long run, must be enough to cover what type of costs? B. less than the cost to leave the plant idle and greater than the cost to buy the boxes.
A. Designing costs C. Servicing costs C. greater than the cost to leave the plant idle and lower than the cost to buy boxes from a

15
Incremental Analysis

supplier. desired profit.


D. less than both the cost to leave the plant idle and the cost to make or buy the boxes.
62. Which of the following is NOT a cost concept commonly used in applying the cost-plus
Minimum acceptable price approach to product pricing?
With excess capacity A. Total cost concept. C. Variable cost concept.
55. If there is excess capacity, the minimum acceptable price for a special order must cover B. Product cost concept. D. Fixed cost concept.
A. variable costs associated with the special order.
B. variable and fixed manufacturing costs associated with the special order. 63. The cost-plus pricing formula that takes into consideration all costs -- fixed, variable, and
C. variable and incremental fixed costs associated with the special order. manufacturing, as well as selling and administrative costs -- is called the percentage of
D. variable costs and incremental fixed costs associated with the special order plus the A. full costs. C. total variable costs.
contribution margin usually earned on regular units. B. variable manufacturing costs. D. absorption costs.

At full capacity Target pricing


56. If a firm is at full capacity, the minimum special order price must cover 43. The concept of target pricing is employed when:
A. variable costs associated with the special order. A. a company wishes to set price in order to capture a predetermined market share.
B. variable and fixed manufacturing costs associated with the special order. B. a price is pre-set by market conditions.
C. variable and incremental fixed costs associated with the special order. C. a company wishes to meet marketing goals.
D. variable costs and incremental fixed costs associated with the special order plus foregone D. All of the above.
contribution margin on regular units not produced.
Target cost approach
Product life cycle 61. In contrast to the total product and variable cost concepts used in setting seller's prices, the
45. A product life cycle includes the phases of target cost approach assumes that:
A. research and development and design C. marketing and distribution A. a markup is added to total cost. C. a markup is added to variable cost.
B. purchasing and production D. all of the above B. selling price is set by the marketplace. D. a markup is added to product cost.

Product pricing Sell-as-is-or-process further


Variable cost approach Joint products
60. Managers who often make special pricing decisions are more likely to use which of the 67. Two or more manufactured products that have significant sales values and are not uniquely
following cost concepts in their work? identifiable as individual products until the split-off point are called
A. Total cost. C. Variable cost. A. common products. C. co-mingled products.
B. Product cost. D. Fixed cost. B. joint products. D. cooperative products.

Cost-plus approach Relevant costs


59. In using the variable cost concept of applying the cost-plus approach to product pricing, what Incremental revenue
is included in the markup? 32. Incremental revenue is:
A. Total costs plus desired profit. A. a difference in costs between two decisions.
B. Desired profit. B. a concession based on competitive influences.
C. Total selling and administrative expenses plus desired profit. C. additional revenue across decision choices from potential sales.
D. Total fixed manufacturing costs, total fixed selling and administrative expenses, and D. the difference between selling price and variable costs.

16
Incremental Analysis

D. decrease in direct fixed costs.


Cost to process further
65. Which of the following costs is relevant in deciding whether to sell joint products at split-off or Irrelevant cost
process them further? 80. Which of the following should not enter into decision of whether to drop product?
A. The unavoidable costs of further processing. A. Unavoidable costs
B. The additional costs of further processing. B. Avoidable costs
C. The variable costs of operating the joint process. C. Revenue that would be lost
D. The cost of materials used to make the joint products. D. Nonfinancial impacts of the decision

68. What are the manufacturing costs incurred beyond the split-off point called? Decision rule
A. Separable costs. C. Severance costs. 79. As long as its marginal cost is lower than its marginal revenue, a company should
B. Joint costs. D. Common costs. A. suspend additional production and sales activities.
B. perform a cost-benefit balance analysis before producing and selling additional products.
Decision rule C. engage in additional production and sales activities.
64. How does a company determine whether to sell a product as is or process it further? D. examine cost behaviors and develop a cost function to measure the cost of future
A. If the costs to process further exceed the costs of current production, the product should production.
be sold as is.
B. If the costs to process further exceed the costs of current production, the product should Short-run profit maximization
be processed further. Factors affecting sales mix
C. If the increase in revenue from selling the product after further processing is greater than 70. Which of the following is an important factor affecting the sales mix of any company?
the additional costs incurred in further processing, the company should opt for further A. organizational advertising expenditures
processing. B. organizational sales force compensation plan
D. If the revenues generated by processing the product further exceed the revenues from C. product selling price
selling the product as is, the company should process further. D. All of the above

Keep-or-drop decision To relax a constraint


Strategic considerations 73. Which of the following will relax a constraint?
66. The decision to keep or drop products or services involves strategic consideration of the: A. Outsourcing all or part of the bottleneck operation
A. potential impact on remaining products or services B. Working overtime at the bottleneck operation
B. impact on employee morale C. Retraining employees and shifting them from the bottleneck
C. growth potential of the firm D. A and B, only
D. All of the above answers are correct
Decision rule
Goal 76. A product mix decision involves
78. The goal in deciding whether to add or drop products, services, or departments is to obtain the A. Influencing the sales volume mix of the products to minimize cost.
greatest B. Influencing the sales volume mix of the products to maximize revenue.
A. reduction in total costs. C. Producing the maximum amount of items that provide the highest contribution margin.
B. contribution possible to cover unavoidable costs. D. Producing the maximum amount of items that carry the lowest per-unit cost.
C. increase in sales revenues.

17
Incremental Analysis

71. A useful device for solving production problems involving multiple products and limited A. its failure to recognize fixed costs.
resources is: B. its failure to recognize depreciation expense.
A. gross sales per unit of product C. net profit per unit of product C. its inability to control waste.
B. contribution per unit of scarce resource D. total benefit D. its inability to recognize financing costs of the production in question.

72. When there is only one production constraint and excess demand, it is generally best to focus PROBLEMS:
production and sales on the product with the highest: Incremental (decremental) cost
1
A. Contribution per unit of scarce resource C. Contribution margin in pesos . For the year ended April 30, 2007, Salmo Company incurred direct costs of P800,000 based
B. Margin of Safety D. Operating Leverage on a particular course of action. Had a different course of action been taken, direct costs
would have been P650,000. In addition, Salmos fixed costs during the fiscal year were
69. When there is one scarce resource, the product that should be produced first is the product P110,000.
with the highest The incremental (decremental) cost was:
A. contribution margin per unit of the scarce resource. A. P 40,000 C. P 150,000
B. sales price per unit of scarce resource. B. P( 40,000) D. P(150,000)
C. demand.
D. contribution margin per unit. Opportunity cost
2
. Luzon Fabricators, Inc. estimates that 60,000 special components will be used in the
74. Uranus Company has 2 products that use the same manufacturing facilities and cannot be manufacture of a specialty steel window for the whole next year. Its supplier quoted a price of
subcontracted. Each product has sufficient orders to utilize the entire manufacturing capacity. P60 per component. Luzon prefers to purchase 5,000 units per month, but its supplier could
For short-run profit maximization, Uranus should manufacture the product with the not guarantee this delivery schedule. In order to ensure availability of these components,
A. Lower total manufacturing costs for the manufacturing capacity. Luzon is considering the purchase of all the 60,000 units at the beginning of the year.
B. Lower total variable manufacturing costs for the manufacturing capacity. Assuming Luzon can invest cash at 8%, the companys opportunity cost of purchasing all the
C. Greater gross profit per hour of manufacturing capacity. 60,000 units at the beginning of the year is
D. Greater contribution margin per hour of manufacturing capacity. A. P132,000 C. P144,000
B. P150,000 D. P264,000
75. Profit can be maximized by producing products with the highest
A. selling price Defective/obsolete inventory
B. contribution margin Incremental net income
3
C. contribution margin per unit of items that are best sellers . Sieney & Company has 24,000 defective units of a product that cost P8 per unit to
D. contribution per unit of the constraining resource manufacture, and can be sold for P4 per unit. These units can be reworked for P2 per unit and
sold at their full price of P12 each. If Sieney reworks the defective units, how much
77. A company should advertise those products that incremental net income will result?
A. Require the lowest commitment of resources to produce A. P144,000 C. P 72,000
B. Have the largest total contribution margin B. P 96,000 D. P 48,000
C. Can be outsourced
D. Have the largest total contribution margin after deducting the cost of the ad campaign Minimum price
4
. Joji Company manufactures and sell FM radios. Information on last years operations (sales
Pitfall and production of the 2006 model) follows:
81. The major pitfall in the contribution margin approach to pricing is Selling price P300

18
Incremental Analysis

Cost per unit: The special order requires 1,000 kilograms of powdered Nitrocide, a solid chemical regularly
Direct materials 70 used in the companys products. The current stock of Nitrocide is 8,000 kilograms at a book
Direct labor 40 value of P8.10 per kilogram. If the special order is accepted, the firm will be forced to restock
Overhead (50% variable) 60 powdered Nitrocide earlier than expected, at a predicted cost of P8.70 per kilogram. Without
Selling costs (40% variable) 100 the special order, the purchasing manager predicts that the price will be P8.30, when normal
Production in units 10,000 restocking takes place. Any order of the Nitrocide must be in 5,000 kilograms.
Sales in units 9,500 What is the relevant cost of powdered Nitrocide to be included in the special order?
At this time (May 2007), the 2007 model is in production and it renders the 2006 model radio A. P 8,700 C. P10,300
obsolete. A foreign firm is willing to purchase the obsolete products at a net price of P140 B. P 8,300 D. P43,500
each. If the remaining 500 units of the 2006 model radios are to be sold through regular
channels, what is the minimum price the company would accept for the radios? Incremental cost
8
A. P300 C. P270 . Balagtas & Company expects to incur the following costs at the planned production level of
B. P180 D. P 40 10,000 units:
Direct materials P100,000
Special order Direct labor 120,000
Unit relevant cost Variable overhead 60,000
5
. Venus Company, a manufacturer of lamps, budgeted sales of 400,000 lamps at P20 per unit Fixed overhead 30,000
for the year. Variable manufacturing costs were budgeted at P8 per unit, and fixed The selling price is P50 per unit. The company currently operates at full capacity of 10,000
manufacturing costs at P 5 per unit. A special order offering to buy 40,000 lamps for P11.50 units. Capacity can be increased to 13,000 units by operating overtime. Variable costs
each was received by Venus in April. Venus has sufficient plant capacity to manufacture the increase by P14 per unit for overtime production. Fixed overhead costs remain unchanged
additional quantity of lamps; however, the production would have to be done by the present when overtime operations occur. Balagtas has received a special order from Florante, Inc.
work force on an overtime-basis at an estimated additional cost of P1.50 per lamp. Venus who has offered to buy 2,000 units at P45 each.
will not incur any selling expenses as a result of the special order. Venus Company would What is the incremental cost associated with this special order?
have a unit relevant cost of A. P42,000 C. P31,000
A. P 8.00 C. P 9.50 B. P84,000 D. P62,000
B. P13.00 D. P14.50
Minimum acceptable price
6 9
. Wawa Enterprises has the capacity to produce 10,000 bearings, but operates at 90% of . Brace Co. has considerable excess manufacturing capacity. A special job orders cost sheet
capacity. Bearings normally sell for P60 each, and cost an average of P50 to make, including includes the following applied manufacturing overhead costs:
a share of the monthly fixed costs of P180,000. Ilog Corp has offered to buy 1,000 bearings at Variable costs P56,250
P40 each. What is the relevant cost per unit? Fixed costs 45,000
A. P 20 C. P 40 The fixed costs include a normal P6,800 allocation for in-house design costs, although no in-
B. P 30 D. P 50 house design will be done. Instead, the special job will require the use of external designers
costing P13,750.
Total relevant cost What is the minimum acceptable price for the job?
7
. Intellectual Co. recently received an order for a product that it does not normally produce. A. P 63,050 C. P101,250
Since the company has excess production capacity, management is considering accepting the B. P 70,000 D. P108,200
order. In analyzing the decision, the assistant controller is compiling the relevant costs of
10
producing the order. . The cost to produce 24,000 units at 70% capacity consists of:

19
Incremental Analysis

13
Direct materials P360,000 . De Silva Co. is a manufacturer of industrial components. One of their products that is used as
Direct labor 540,000 a subcomponent in auto manufacturing is KB69. This product has the following financial
Factory overhead, all fixed 290,000 structure per unit:
Selling expense (35% variable, 65% fixed) 240,000 Selling price P150
What unit price would the company have to charge to make P22,500 on a sale of 1,500 Direct materials P 20
additional units that would be shipped out of the normal market area? Direct labor 15
A. P 51 C. P 41 Variable manufacturing overhead 12
B. P 56 D. P 50 Fixed manufacturing overhead 30
Variable shipping and handling 3
11
. Kaila Companys unit cost of manufacturing and selling a given item at an activity level of Fixed selling and administrative 10
10,000 units per month are: Total P 90
Manufacturing costs De Silva is operating at full capacity. It has received a special, one-time, order for 1,000 KB69
Direct materials P39 parts. The next best alternative use of the excess capacity is to produce LB46, resulting in a
Direct labor 6 contribution margin of P10,000. The minimum price that is acceptable for this one-time special
Variable overhead 8 order is
Fixed overhead 9 A. P 60 C. P 70
Selling expenses B. P 87 D. P100
Variable 30
14
Fixed 11 . Sylvania Company. is currently operating at a loss of P15,000. The sales manager has
The company desires to seek an order for 5,000 units from a foreign customer. The variable received a special order for 5,000 units of product, which normally sells for P35 per unit. Costs
selling expenses will be reduced by 40%, but the fixed costs for obtaining the order will be associated with the product are: direct material, P6; direct labor, P10; variable overhead, P3;
P20,000. Domestic sales will not be affected by the order. applied fixed overhead, P4; and variable selling expenses, P2. The special order would allow
The minimum break-even price per unit to be considered on this special sale is the use of a slightly lower grade of direct material, thereby lowering the price per unit by P1.50
A. P 71 C. P 69 and selling expenses would be decreased by P1. If Sylvania wants this special order to
B. P 75 D. P 84 increase the total net income for the firm to P25,000, what sales price must be quoted for each
of the 5,000 units?
12
. Chrisy Company sells a product for P18 per unit and the standard cost card for the product A. P18.50 C. P29.00
shows the following costs: B. P24.50 D. P26.50
Direct materials P 1.00
Direct labor 2.00 Maximum lost regular sales
15
Overhead (80% fixed) 7.00 . Chua Company sells a product for P20 with variable cost of P8 per unit. Chua could accept a
Total P10.00 special order for 1,000 units at P14. If Chua accepted the order, how many units could it lose
Chrisy received a special order for 1,000 units of the product. The only additional cost to at the regular price before the decision become unwise?
Chrisy would be foreign import taxes of P1 per unit. If Chrisy is able to sell all of the current A. 1,000 units C. 500 units
production domestically, what would be the minimum sales price that Chrisy would consider for B. 200 units D. 0 units
this special order?
16
A. P 18 C. P 17 . Filamer Company currently sells 1,000 units of product M for P2 each. Variable costs are
B. P 19 D. P 11 P1.50. A discount store has offered P1.70 per unit for 400 units of product M. The managers
believe that if they accept the special order, they will lose some sales at the regular price.

20
Incremental Analysis

Determine the number of units they could lose before the order become unprofitable. production of 30,000 units.
A. 200 units. C. 400 units. Direct materials P 4
B. 160 units. D. 500 units Direct labor 12
Variable manufacturing overhead 6
Effect on profit of accepting the order Fixed manufacturing overhead 8
17
. You have been approached by a foreign customer who wants to place an order for 15,000 The company has the capacity to produce 40,000 units. The product regularly sells for P40. A
units of Product C at P22.50 a unit. You currently sell this item for P39 a unit, and the item wholesaler has offered to pay P32 a unit for 2,000 units.
has a cost of P29 a unit. Further analysis reveals that you will not be paying sales commission If the firm accepts the special order the effect on its operating income would be a
of P2.50 a unit on this sales and its packaging requirement will save you an additional P1.50 A. P20,000 increase C. P4,000 increase
per unit. However, the additional graphics required on this job will cost you P30,000. Note also B. P16,000 decrease D. P 0 effect
that fixed costs amounting to P400,000 for the production of 50,000 of such products by the
21
firm will not change. You decide to accept this order, but another customer who buys an . Louderhead Company makes bull-repellent scent according to a traditional Western recipe,
average of 2,000 units for the period wants to pay you P22.50 rather than the regular price of which normally sells at P90 per unit. Normal production volume is 10,000 ounces per month.
P39 a unit. Profit will Average cost is P50 per ounce, of which P20 is direct material and P10 is variable conversion
A. increase profit by P19,500 C. increase profit by P52,500 cost. This product is seasonal. After July, demand for this product drops to 6,000 ounces
B. increase profit by P16,500 D. decrease profit by P52,500 monthly. In November, Garrison Co. offers to buy 1,500 ounces for P60,000. If Louderhead
accepts the order, it must design a special label for Garrison at a cost of P5,000. Each label
18
. The Thermo Company has received a special order for 300 units of product X for P6 a unit. It will cost P2.50 to make and apply. Louderhead should:
usually sells for P9.50 a unit with a cost of P7.50 a unit inclusive of 75 cents a unit as sales A. accept the order, at a gain of P6,250
commission that will not be paid on this order. The cost also includes P3 in manufacturing B. reject the order, at a loss of P18,750
overhead, was two-third of which is for the fair share of depreciation, rent, utilities and C. reject the order, at a loss of P23,750
supervisor's salary. The latters (supervisor's salary) accounts for one-half of this amount. D. accept the order, at a gain of P11,250
Assuming that excess capacity is available, and this order requires a mold that costs P150,
accepting the order will increase Question Nos. 68 and 69 are based on the following information:
A. loss by P225 C. gain by P225 The Disk Division of Systems Specialist Company produces a high quality computer disks. Unit
B. loss by P375 D. gain by P375 production costs (based on capacity production of 100,000 units per year) follow:
Direct materials P50
19
. Alejar Company manufactures a product with a unit variable cost of P50 and a unit sales price Direct labor 20
of P88. Fixed manufacturing costs were P240,000 when 10,000 units were produced and sold. Overhead (20% variable) 10
The company has a one-time opportunity to sell an additional 3,000 units at P70 each in a Other information:
foreign market. This special sale would not affect its present sales. If the company has Sales price 100
sufficient capacity to produce the additional units, acceptance of the special order would affect SG & A costs (40% variable) 15
net income as follows: The Disk Division is operating at a level of 70,000 chips per year.
A. Income would decrease by P 12,000.
22
B. Income would increase by P 12,000. . What is the minimum price that the division would consider on a special order of 1,000 disks
C. Income would increase by P210,000. to be distributed through normal channels?
D. Income would increase by P 60,000. A. P 72 C. P 81
B. P 78 D. P 6
20
. KC Industries manufactures a product with the following costs per unit at the expected

21
Incremental Analysis

23
. Assuming that that the Disk Division is producing and selling at capacity. What is the minimum capacity costs that will not be affected by any make-or-buy decision represent 60% of the
selling price that the division would consider on a special order of 1,000 chips on which no applied overhead.
variable period costs would be incurred? The available 30,000 machine hours are to be scheduled so that ELM realizes maximum
A. P100 C. P 94 potential cost savings. The relevant unit production costs that should be considered in the
B. P 72 D. P 90 decision to schedule machine time are:
A. P54.00 for Beta and P147.00 for Zeta C. P14.00 for Beta and P127.00 for Zeta
Make-or-buy decision B. P50.00 for Beta and P150.00 for Zeta D. P30.00 for Beta and P135.00 for Zeta
Relevant costs
24
. For the past 12 years, the JLO Company has produced the small electric motors that fit into its Maximum buy price
26
main product line of dental drilling equipment. As materials costs have steadily increased, the . The following are a companys monthly unit costs to manufacture and market a particular
controller of the JLO Company is reviewing the decision to continue to make the small motors product.
and has identified the following facts: Manufacturing Costs:
1) The equipment which is used to manufacture the electric motors has a book Direct materials P2.00
value of P1,500,000. Direct labor 2.40
2) The space being occupied now by the electric motor manufacturing department Variable indirect 1.60
could be used to eliminate the need for storage space which is presently being rented. Fixed indirect 1.00
3) Comparable units can be purchased from an outside supplier for P597.50.
4) Four of the people who work in the electric motor manufacturing department Marketing Costs:
would be terminated and given eight weeks of separation pay. Variable 2.50
5) A P750,000 unsecured note is still outstanding on the equipment that is being Fixed 1.50
used in the manufacturing process. The company must decide to continue making the product or buy it from an outside supplier.
Which of the items above are relevant to the decision that the controller has to make? The supplier has offered to make the product at a level of quality that the company prescribes.
A. 1, 2, 4, and 5 C. 1, 3, 4, and 5 Fixed marketing costs would be unaffected, but variable marketing costs would continue at
B. 1, 3, and 4 D. 2, 3, and 4 30% if the company were to accept the proposal.
What is the maximum amount per unit that the company can pay the supplier without
Relevant cost to make decreasing its operating income?
25
. ELM Electronics has the following standard costs and other data: A. P8.50 C. P7.75
Part Beta Part Zeta B. P6.75 D. P5.25
Direct materials P 4.00 P80.00
27
Direct labor 10.00 47.00 . Sinta Company can make 1,000 units of a necessary component with the following costs:
Factory overhead 40.00 20.00 Direct Materials P64,000
Unit standard cost P54.00 P147.00 Direct Labor 16,000
Units needed per year 6,000 8,000 Variable Overhead 8,000
Machine hours per unit 4 2 Fixed Overhead ?
Unit cost if purchased P50.00 P150.00 The company can purchase the 1,000 units externally for P104,000. An analysis shows that at
In the past years, ELM has manufactured all of its required components; however, this year this external price, the company is indifferent between making or buying the part. Sinta
only 30,000 hours of otherwise idle machine time can be devoted to the production of Company could avoid P6,000 in fixed overhead costs if it acquires the components externally.
components. Accordingly, some of the parts must be purchased from outside suppliers. In If cost minimization is the major consideration and the company would prefer to buy the
producing the parts, factory overhead is applied at P10 per standard machine hour. Fixed components, what is the maximum external price that Sinta Company would accept to acquire

22
Incremental Analysis

the 1,000 units externally? Effect of buy decision


A. P102,000. C. P 96,000. On fixed overhead cost
31
B. P 94,000. D. P 88,000. . Sisa's Shop can make 1,000 units of a necessary component with the following costs:
Direct Materials P64,000
28
. Almeda's Shop can make 1,000 units of a necessary component with the following costs: Direct Labor 16,000
Direct Materials P64,000 Variable Overhead 8,000
Direct Labor 16,000 Fixed Overhead ?
Variable Overhead 8,000 The company can purchase the 1,000 units externally for P104,000. The unavoidable fixed
Fixed Overhead ? costs are P5,000 if the units are purchased externally. An analysis shows that at this external
The company can purchase the 1,000 units externally for P104,000. None of Almeda price, the company is indifferent between making or buying the part. What are the fixed
Company's fixed overhead costs can be reduced, but another product could be made that overhead costs of making the component?
would increase profit contribution by P16,000 if the components were acquired externally. If A. P21,000. C. P11,000.
cost minimization is the major consideration and the company would prefer to buy the B. P16,000. D. Cannot be determined.
components, what is the maximum external price that Almeda Company would be willing to
accept to acquire the 1,000 units externally? On income
32
A. P 86,000. C. P 96,000. . Sylvan Processing Company is considering whether to make 2,000 units of product Whirl
B. P110,000. D. P104,000. which costs P16 a unit or buy it from outside for P15 a unit. A further analysis shows that if
product Whirl is outsourced, fixed costs of P8,000 attributable to this product will be reduced
Effect of make decision by 25%.
29
. A business is operating at 90% of capacity and is currently purchasing a part which is being If the product is outsourced, Sylvan will
used in its manufacturing operations for P15 per unit. The unit cost for the business to make A. Decrease profit by P2,000 C. Increase profit by P2,000
the part is P20, including fixed costs, and P12, not including fixed costs. If 30,000 units of the B. Decrease profit by P4,000 D. Increase profit by P4,000
part are normally purchased during the year but could be manufactured using unused
33
capacity, what would be the amount of differential cost, increase or decrease, from making the . Sylvan Processing Company is considering whether to make 2,000 units of product Whirl
part rather than purchasing it? which costs P16 a unit or buy it from outside for P15 a unit. A further analysis shows that if
A. P150,000 cost increase C. P150,000 cost decrease product Whirl is outsourced, fixed costs of P8,000 attributable to this product will be reduced
B. P 90,000 cost decrease D. P 90,000 cost increase by 25%. If Sylvan Processing Company purchased the product Whirl, the space could be
rented out for P6,000. If the product is outsourced, profit would
30
. Alfaro's Manufacturing Company can make 100 units of a necessary component part with the A. decrease, P2,000 C. increase, P2,000
following costs: B. decrease, P4,000 D. increase, P4,000
Direct Materials P80,000
34
Direct Labor 13,000 . It costs P450,000 to make 15,000 units of a part in this plant. This cost includes material of
Variable Overhead 40,000 P90,000, direct labor of P120,000, variable overhead of P15,000, and P225,000 in fixed
Fixed Overhead 27,000 overhead inclusive of P45,000 in depreciation and common overhead allocation of P150,000.
If Alfaro's Manufacturing Company can purchase the component externally for P145,000 and The balance is for the section supervisor's salary. The part can be purchased for P20 a unit. If
only P4,000 of the fixed costs can be avoided, what is the correct make or buy decision? the part is purchased, the space released can be rented for P65,000. If the part is purchased,
A. Make and save P8,000 C. Make and save P20,000 the company will
B. Buy and save P8,000 D. Buy and save P20,000 A. lose P20,000 C. gain P20,000
B. lose P45,000 D. gain P45,000

23
Incremental Analysis

Garland, the unit cost of WS73 would


35
. Lane Co. manufactures ballpoint pens. Another manufacturer has offered to supply Lane with A. Increase by P4,800 C. Decrease by P6,200
the 5,000 ink cartridges that it needs annually. The cost to buy the cartridges would be P15 B. Decrease by P3,200 D. Increase by P1,800
each. In producing its own cartridges, Lane has incurred P10 in fixed costs and P8 in variable
38
costs. If Lane buys the cartridges, its net income will: . The Rural Cooperative, Inc. produces 1,000 units of Part M per month. The total
A. not change C. increase by P35,000 manufacturing costs of the part are as follows:
B. decrease by P35,000 D. increase by P25,000 Direct materials P10,000
Direct labor 5,000
36
. The Rainbow Company manufactures Part No. 498 for use in its production cycle. The cost Variable overhead 5,000
per unit if 20,000 units of Part No. 498 are manufactured are as follows: Fixed overhead 30,000
Direct materials P6 Total manufacturing cost P50,000
Direct labor 30 An outside supplier has offered to supply the part at P30 per unit. It is estimated that 20% of
Variable overhead 12 the fixed overhead being assigned to Part M will no longer be incurred if the company
Fixed overhead applied 16 purchases the part from the outside supplier.
Total unit cost P64 If Rural Cooperative purchases 1,000 units of Part M from the outside supplier, its monthly
The Reeves Company has offered to sell 20,000 units of part No. 498 to Rainbow for P60 per operating income will
unit. Rainbow will make the decision to buy the part from Reeves if there is a savings of A. decrease by P 4,000 C. increase by P 1,000
P25,000 for Rainbow. If Rainbow accepts Reevess offer, P9 per unit of the fixed overhead B. decrease by P20,000 D. increase by P20,000
applied would be totally eliminated. Furthermore, Rainbow has determined that the released
39
facilities could be used to save relevant costs in the manufacture of part No. 575. In order to . Migs Corporation currently manufactures all component parts used in the manufacture of
have a savings of P25,000, the amount of the relevant costs that would be saved by using the various hand tools. A steel handle is used in three different tools. The budgeted costs per unit
released facilities in the manufacture of Part No. 575 would have to be based on 20,000 units are:
A. P 80,000 C. P125,000 Direct material P6.00
B. P 85,000 D. P140,000 Direct labor 4.00
Variable overhead 1.00
37
. Leis Manufacturing Co. uses 10 units of Part Number WS73 each month in the production of Fixed overhead 2.00
computer printer. The unit cost to manufacture one unit of WS73 is presented below. Total unit cost P13.00
Direct materials P 1,000 Sans Steel, Inc. has offered to supply 20,000 units of the handle to Migs for P12.50 each
Materials handling (20% of direct material cost) 200 delivered. If Migs currently has idle capacity that cannot be used, accepting the offer will
Direct labor 8,000 A. Decrease the handle unit cost by P0.50.
Manufacturing overhead (150% of direct labor) 12,000 B. Increase the handle unit cost by P1.50.
Total manufacturing cost P21,200 C. Decrease the handle unit cost by P1.50.
Material handling represents the direct variable costs of the Receiving Department that are D. Increase the handle unit cost by P0.50.
applied to direct materials and purchased components on the basis of their cost. This is a
40
separate charge in addition to manufacturing overhead. Leis annual manufacturing overhead . The Minolta, Inc. produces 1,000 units of Part M per month. The total manufacturing costs of
budget is one-third variable and two-thirds fixed. Garland Company, one of Leis reliable the part are as follows:
vendors, has offered to supply part WS73 at a unit price of P15,000. Direct materials P10,000
If Leis purchases the WS73 units from Garland, the capacity being used by Leis to Direct labor 5,000
manufacture these parts would be idle. Should Leis decide to purchase the parts from Variable overhead 5,000

24
Incremental Analysis

Fixed overhead 30,000 Blade Division cannot sell any additional products to outside customers.
Total manufacturing cost P50,000 Should Dana allow its Lawn Products Division to purchase the blades from the outside
An outside supplier has offered to supply the part at P30 per unit. It is estimated that 20% of supplier, and why?
the fixed overhead assigned to Part M will no longer be incurred if the company purchases the A. Yes, because buying the blades would save Dana Company P500.
part from the outside supplier. B. No, because making the blades would save Dana Company P1,500.
If Minolta purchases 1,000 units of Part M from the outside supplier per month, then its C. Yes, because buying the blades would save Dana Company P2,500.
monthly operating income will D. No, because making the blades would save Dana Company P2,500
A. decrease by P 4,000 C. increase by P 1,000
43
B. decrease by P20,000 D. increase by P20,000 . The Connell Company uses 5,000 units of Part 501 each year. The cost of manufacturing one
unit Part 501 at this volume is as follows:
41
. Bulacan Company manufactures part G for use in the production of its principal product. The Direct materials P2.50
costs per unit for 10,000 units of part G are as follows: Direct labor 3.50
Direct materials P 3 Variable overhead 1.50
Direct labor 15 Fixed overhead 1.00
Variable overhead 6 Total P8.50
Fixed overhead 8 An outside supplier has offered to sell Connell unlimited quantities of Part 501 at a unit cost of
Total P32 P7.75. If Connell accepts this offer, it can eliminate 50 percent of the fixed costs assigned to
Pampanga Company has offered to sell Bulacan 10,000 units of part G for P30 per unit. If part 501. Furthermore, the space devoted to the manufacture of Part 501 would be rented to
Bulacan accepts Pampangas offer, the released facilities could be used to save P45,000 in another company for P6,000 per year. If Connell accepts the offer of the outside supplier,
relevant costs in the manufacture of part H. In addition, P5 per unit of the fixed overhead annual profits will
applied to part G would continue. A. Increase by P13,500 C. Increase by P 7,250
What alternative is more desirable and by what amount? B. Increase by P11,000 D. Increase by P 1,250
A. B. C. D.
Alternative Manufacture Manufacture Buy Buy Point of indifference - Units
44
Amount P10,000 P15,000 P15,000 P10,000 . Mars Industries is a multi-product company that currently manufactures 30,000 units of Part
QS42 each month for use in the production of its main product. The facilities now being used
. Blade Division of Dana Company produces hardened steel blades. One-third of the Blades
42 to produce Part QS42 have fixed monthly cost of P150,000 and a capacity to produce 84,000
Divisions output is sold to the Lawn Products Division of Dana; the remainder is sold to units per month. If Mars were to buy Part QS42 from an outside supplier, the facilities would
outside customers. The Blade Divisions estimated sales and standard costs data for the fiscal be idle, but 60 percent of its fixed costs would not continue. The variable production costs of
year ending June 30 are as follows: Part QS42 are P11 per unit.
Lawn Products Outsiders If Mars Industries is able to obtain Part QS42 from an outside supplier at a unit purchase price
of P12.875, the monthly usage at which it will be indifferent between purchasing and making
Sales P15,000 P40,000
Part QS42 is
Variable costs (10,000) (20,000)
A. 30,000 units C. 80,000 units
Fixed costs (3,000) (6,000)
B. 32,000 units D. 48,000 units
Gross margin P 2,000 14,000
Unit sales 10,000 20,000 Point of indifference - price
The Lawn Products Division has an opportunity to purchase 10,000 identical quality blades 45
. Calero Manufacturing Company can make 100 units of a necessary component part with the
from an outside supplier at a cost of P1.25 per unit on a continuing basis. Assume that the following costs:

25
Incremental Analysis

Direct Materials P80,000 determined as follows:


Direct Labor 13,000 Product X Product Y
Variable Overhead 40,000 Revenue P130 P80
Fixed Overhead 27,000 Variable costs 70 P38
If Calero Manufacturing Company purchases the component externally, P20,000 of the fixed Contribution margin P 60 P42
costs can be avoided. At what external price for the 100 units is the company indifferent Total demand for X is 16,000 units and for Y is 8,000 units. Machine hour is a scarce
between making or buying? resource. 42,000 machine hours are available during the year. Product X requires 6 machine
A. P160,000. C. P153,000. hours per unit while product Y requires 3 machine hours per unit.
B. P113,000. D. P133,000. How many units of X and Y should Hingis Corporation produce?
A. B. C. D.
Profit maximization
Product X 16,000 8,000 7,000 3,000
Point of indifference
46 Product Y zero 4,000 zero 8,000
. Dipsum Soft Drinks makes three products: iced tea, soda, and lemonade. The following data
are available: 49
. Mary Manufacturing has assembled the following data pertaining to two popular products.
Iced Tea Soda Lemonade
Blender Electric mixer
Sales price per unit P9.00 P6.00 P5.00
Direct materials P 6 P11
Variable cost per unit 3.00 1.50 1.00 Direct labor 4 9
Contribution margin per unit P6.00 P4.50 P4.00 Factory overhead @ P16 per hour 16 32
Dipsum is experiencing a bottleneck in one of its processes that affects each product as Cost if purchased from an outside supplier 20 38
follows: Annual demand (units) 20,000 28,000
Iced Tea Soda Lemonade Past experience has shown that the fixed manufacturing overhead component included in the
Bottleneck process hours per unit 3 3 4 cost per machine hour averages P10. Mary has a policy of filling all sales orders, even if it
What price for lemonade would equate its profitability to that of soda? means purchasing units from outside suppliers.
A. P8.00. C. P6.00. If 50,000 machine hours are available, and Mary Manufacturing desires to follow an optimal
B. P7.00. D. P5.50. strategy, it should
A. produce 25,000 electric mixers, and purchase all other units as needed
Optimal mix B. produce 20,000 blenders and 15,000 electric mixers, and purchase all other units as
47
. Product A sells for P12 per unit and its variable cost per unit is P10. Product B sells for P15 needed
per unit and its variable cost per unit is P12. The plant capacity is 350,000 machine hours and C. produce 20,000 blenders and purchase all other units as needed
both products require one machine hour to manufacturer. Which of the following will provide D. produce 28,000 electric mixers and purchase all other units as needed
the best sales mix of Product A and Product B assuming the market limitation of Product A is
200,000 units and the market limitation of Product B is 250,000 units? Decision
A. 250,000 units of Product A, 100,000 units of Product B 50
. A company can sell all the units it can produce of either Product A or Product B but not both.
B. 50,000 units of Product A, 300,000 units of Product B Product A has a unit contribution margin of P36 and takes two machine hours to make and
C. 100,000 units of Product A, 250,000 units of Product B Product B has a unit contribution margin of P45 and takes three machine hours to make. If
D. 150,000 units of Product A, 200,000 units of Product B there are 1,000 machine hours available to manufacture a product, income will be
A. P3,000 more if Product A is made. C. P3,000 less if Product A is made.
48
. The Hingis Corporation manufactures two products: X and Y. Contribution margin per unit is B. P3,000 less if Product B is made. D. the same if either product is made.

26
Incremental Analysis

C. 6,000 units of Plastic and manufacturing the remaining bearings.


51
. The Baco Company produces three products with the following costs and D. 5,000 units of Metal and manufacturing the remaining bearings.
selling prices:
53
A B C . HILO Company manufactures electric carpentry tools. The production department had met all
Selling price per unit P16 P21 P21 production requirements for the current month and has an opportunity to produce additional
Variable cost per unit 7 11 13 units of product with its excess capacity. Unit selling prices and unit costs for three different
Contribution margin per unit P 9 P10 P 8 drill models are as follows:
Direct labor hours per unit 1.0 1.5 2.0 Home Model Deluxe Model Pro Model
Machine hours per unit 4.5 2.0 2.5 Selling price P58 P65 P80
In what order should the three products be produced if either the direct labor-hours or the Direct material 16 20 19
machine hours are the companys production constraint? Direct labor (P10 per hour) 10 15 20
A. B. C. D. Variable overhead 8 12 16
Direct labor hours A, B, C B, C, A B, C, A A, B, C Fixed overhead 16 5 15
Machine hours B, C, A B. C. A A, C, B A, C, B Variable overhead is applied on the basis of direct-labor pesos, while fixed overhead is applied
on the basis of machine hours. There is sufficient demand for the additional production of any
. Scarce Company has been producing two types of bearings, Plastic and Metal, for its own use
52 model in the product line. If it has excess machine capacity but a limited amount of labor time,
in the production of main products. The data regarding these two bearings follow: to which product or products should HILO Company devote its excess production?
Plastic Metal A. Home model C. Deluxe model
Machine hours required per unit 3.0 4.5 B. Pro Model D. Equally
Standard cost per unit
Prime costs P 8.00 P 9.00 . Product A sells for P12 per unit and its variable cost per unit is P10. Product B sells for P15
54

Variable overhead* 3.00 4.00 per unit and its variable cost per unit is P12. The plant capacity is 350,000 machine hours and
Fixed overhead** 4.50 6.75 Product A requires 48 minutes to complete while Product B requires 75 minutes. Which of the
following will provide the best sales mix of Product A and Product B assuming the market
Total P15.50 P19.75
limitation of Product A is 200,000 units and the market limitation of Product B is 250,000 units?
A. 46,875 units of Product A, 250,000 units of Product B
*Variable manufacturing overhead is applied on the basis of direct labor hours. B. 200,000 units of Product A, 152,000 units of Product B
**Fixed manufacturing overhead is applied on the basis of machine hours. C. 152,000 units of Product A, 200,000 units of Product B
D. 100,000 units of Product A, 250,000 units of Product B
Scarces annual requirements for these bearings is 7,000 units of Plastic and 11,000 units of
Metal. Recently, Scarces management decided to devote additional machine hours to other . Dimasalang Company has only 25,000 hours of machine time each month to manufacture its
55
product lines resulting to only 48,000 machine hours per year that can be dedicated to the two products. Product X has a contribution margin of P50 and Product Y has a contribution
production of the bearings. An outside company has offered to sell Scarce the annual supply margin of P64. Product X requires 5 machine hours and Product Y, 8 hours. If Dimasalang
of the bearings at prices of P15.50 for Plastic and P17.50 for Metal. Scarce wants to schedule wants to dedicate 80% of its machine time to the product that will provide the most income, it
the otherwise idle 48,000 machine hours to produce bearings so that the company can will have a total contribution margin of
minimize its costs (maximize its net benefits). A. P250,000 C. P210,000
Scarce Company will maximize its net benefits by purchasisng B. P240,000 D. P200,000
A. 7,000 units of Plastic and manufacturing the remaining bearings.
B. 11,000 units of Metal and manufacturing 7,000 units of Plastic. Sell-as-is-or-process-further
27
Incremental Analysis

Minimum sales P350,000.


56
. Snow Clean Corporation produces cleaning compounds and solutions for industrial and Which of the following answers is correct?
household use. While most of its products are processed independently, a few are related. A. Sell now
Grit 337, a coarse cleaning powder with many industrial uses, costs P16 a pound to make and B. Finish the product because profits will increase by P25,000
sells for P20 a pound. A small portion of the annual production of this product is retained for C. Finish the product because profits will increase by P12,500
further processing in the Mixing Department, where it is combined with several other D. Finish the product because profits will increase by P10,000
ingredients to form a paste, which is marketed as a silver polish selling for P40 per jar. This
further processing requires pound of Grit 337 per jar. Costs of other ingredients, labor, and Effect of decision
60
variable overhead associated with this further processing amount to P25 per jar. Variable . Ottawa Corporation produces two products from a joint process. Information about the two
selling costs are P3 per jar. If the decision were made to cease production of the silver polish, joint products follows:
P56,000 of Mixing Department fixed costs could be avoided. Snow Clean has limited Product X Product Y
production capacity for Grit 337, but unlimited demand for the cleaning powder. Anticipated production 2,000 lbs 4,000 lbs
What is the minimum number of jars of silver polish that would have to be sold to justify further Selling price per pound at split-off P30 P16
processing of Grit 337. Additional processing costs/pound after split-off P15 P30
A. 8,000 C. 7,000 (all variable)
B. 5,600 D. 4,667 Selling price/pound after further processing P40 P50
The joint cost is P85,000. Ottawa currently sells both products at the split-off point. If Ottawa
Decision makes decision which maximizes profit, its profit will increase by
57
. Beal Company is starting business and is unsure of whether to sell its product assembled or A. P16,000 C. P 4,000
unassembled. The unit cost of the unassembled product is P40 and Beal Company would sell B. P50,000 D. P10,000
it for P90. The cost to assemble the product is estimated at P18 per unit and Beal Company
believes the market would support a price of P116 on the assembled unit. . The cost to manufacture an unfinished unit is P40 (P30 variable and P10 fixed). The selling
61

What is the correct decision using the sell or process further decision rule? price per unit is P50. The company has unused production capacity and has determined that
A. Sell before assembly, the company will be better off by P18 per unit. units could be finished and sold for P65 with an increase in variable costs of 40%. What is the
B. Sell before assembly, the company will be better off by P26 per unit. additional net income per unit to be gained by finishing the unit?
C. Process further, the company will be better off by P26 per unit. A. P 3 C. P10
D. Process further, the company will be better off by P8 per unit. B. P15 D. P12
58
. Sales of 25,000 units at P7.20 per unit are made monthly. The unit cost is P5.90. Incremental Total processing cost
costs of P1.35 per unit to further process the units will result in the 25,000 units being sold for 62
. Matador Manufacturing schedules a weekly production of 15,000 units of
P8.75 each. Which course of action should the company take?
Product M and 30,000 units of N for which P800,000 common variable costs
A. Commit its resources to a different product
B. Sell the units at the current stage of completion
are incurred. These two products can be sold as is or processed further.
C. Do further processing and sell the units at P8.75 Further processing of either product does not delay the production of
D. Do further processing on only one-half of the units subsequent batches of the joint products. Below are some of the information:
M N
59
. Aaron Company produces a product that can be sold for P250,000 at an intermediate stage. If Unit selling price without further processing P25 P19
Aaron finishes the product, they will incur P75,000 of additional material costs and another Unit selling price with further processing P31 P23
P15,000 in labor and overhead costs. When finished, Aaron will be able to sell the product for Total separate weekly variable costs of further processing P100,000 P110,000

28
Incremental Analysis

To maximize Matadors manufacturing contribution margin, the total separate variable costs of Shutdown point
67
further processing that should be incurred each week are . Bulusan Company normally produces and sells 30,000 units of E14 each month. E14 is a
A. P105,000 C. P110,000 small electrical relay used in the automotive industry as a component part in various products.
B. P100,000 D. P210,000 The selling price is P22 per unit, variable costs are P14 per unit, fixed manufacturing overhead
costs total P150,000 per month, and fixed selling costs total P30,000 per month.
Keep-or-drop decision
Analysis Employment-contract strikes in the companies that purchase the bulk of the E14 have caused
63
. A company is deciding whether or not to eliminate a segment of its business. The segment Bulusan Companys sales to temporarily drop to only 9,000 units per month. Bulusan
generates total sales of P104,000, its direct expenses are P22,000, and its indirect expenses Company estimates that the strikes will last for about two months, after which time sales of
are P26,000. Its cost of goods sold is P64,000. Six thousand pesos of the direct expenses E14 should return to normal. Due to the current low level of sales, however, Bulusan
and P8,000 of its indirect expenses are avoidable expenses. Which of the following is not Company is thinking about closing down its own plant during the two months that the strikes
true? are on. If Bulusan Company does close down its plant, it is estimated that fixed manufacturing
A. This segment has a net loss of P8,000. overhead costs can be reduced to P105,000 per month and that fixed selling costs can be
B. This segment's revenue is greater than its avoidable costs. reduced by 10%. Start-up costs at the end of the shutdown period would total P8,000. Since
C. This segment is a good candidate for elimination. Bulusan Company uses just-in-time production method, no inventories are on hand.
D. This segment's avoidable costs are greater than unavoidable costs.
At what level of unit sales for the two-month period should Bulusan Company be indifferent
Effect of drop decision between temporarily closing the plant or keeping it open?
64
. Banahaw Company plans to discontinue a department that has a contribution margin of A. 11,000 C. 10,000
P240,000 and P480,000 in fixed costs. Of the fixed costs, P210,000 can be avoided. The B. 24,125 D. 8,000
effect of this discontinuance on Banahaws overall net operating income would be a(an)
A. decrease of P30,000 C. increase of P30,000 Equipment replacement
68
B. decrease of P10,000 D. increase of P10,000 . MNL Company has an opportunity to acquire a new machine to replace one of its present
machines. The new machine would cost P90,000, have a 5-year life and no estimated
65
. Mina Co. mines three products. Gold Ore sells for P1,000,000 per ton, variable costs are salvage value. Variable operating costs would be P100,000 per year. The present machine
P600,000 per ton, and fixed mining costs are P6,000,000. The segment margin for 2007 was has a book value of P50,000 and a remaining life of 5 years. Its disposal value now is P5,000,
P1,200,000. The management of Mina Co. was considering dropping the mining of Gold Ore. but it would be zero after 5 years. Variable operating costs would be P125,000 per year.
Only one-half of the fixed expenses are direct and would be eliminated if the segment was Ignore income taxes. Considering the 5 years in total, what would be the difference in profit
dropped. If Gold Ore were dropped, net income for Mina Co. would before income taxes by acquiring the new machine as opposed to retaining the present one?
A. Increase by P2,000,000 C. Decrease by P2,000,000 A. P10,000 decrease C. P35,000 increase
B. Increase by P1,200,000 D. Decrease by P1,200,000 B. P15,000 decrease D. P40,000 increase
66
. Agimat Company plans to discontinue a segment with a P32,000 segment margin. Common Lease
69
expenses allocated to the segment amounted to P45,000, of which P20,000 cannot be . Darren Co. is considering disposing an equipment that costs P50,000 and has P40,000 of
eliminated if the segment were closed. The effect of closing down the segment on Agimat accumulated depreciation to date. Darren Co. can sell the equipment through a broker for
Companys before tax profit would be P25,000 less 5% commission. Alternatively, Minton Co. has offered to lease the equipment for
A. P12,000 decrease C. P12,000 increase five years for a total of P48,750. Darren will incur repair, insurance, and property tax expenses
B. P 7,000 decrease D. P 7,000 increase estimated at P10,000. At lease-end, the equipment is expected to have no residual value. The
net differential income from the lease alternative is:

29
Incremental Analysis

73
A. P15,000. C. P25,000. . Due to a strike in its suppliers plant, Adrenal Company is unable to purchase more material for
B. P 5,000. D. P12,500. the production of CADS. The strike is expected to last for two months. Adrenal Company has
enough material on hand to continue to operate at 30% of normal levels for the two months. If
Comprehensive the plant were closed, fixed overhead costs would continue at 60% of their normal level during
Questions 70 through 74 are based on the following information: the two-month period; the fixed selling costs would be reduced by 20% while the plant was
Adrenal Company has a single product called a CAD. The company normally produces and sells closed. How much is the advantage or disadvantage of closing the plant for the two-month
60,000 CADS each year at a selling price of P32 per unit. The companys unit costs at this level of period?
activity are given below: A. Disadvantage, P144,000 C. Disadvantage, P15,000
Direct materials P10.00 B. Advantage, P144,000 D. Advantage, P15,000
Direct labor 4.50
Variable manufacturing overhead 2.30
74
Fixed manufacturing overhead 5.00 . An outside manufacturer has offered to produce CADS for Adrenal Company and to ship them
Variable selling expenses 1.20 directly to Adrenals customers. If Adrenal Company accepts this offer, the facilities that it
Fixed selling expenses 3.50 uses to produce CADS would be idle; however, fixed overhead costs would be reduced by
Total cost per unit P26.50 75% of their present level. Since the outside manufacturer would pay for all the costs of
shipping, the variable selling costs would be only two-thirds of their present amount. What is
70
. Assume that Adrenal Company has sufficient capacity to produce 90,000 CADS each year the unit cost figure that is relevant for comparison to whatever quoted price is received from
without any increase in fixed manufacturing overhead costs. The company could increase its the outside manufacturer?
sales by 25% above the present 60,000 units each year if it were willing to increase the fixed A. P20.95 C. P20.55
selling expenses by P80,000. The increase in income if the production is increased by 25% is B. P21.35 D. P16.80
A. P130,000 C. P 25,000
B. P108,333 D. P 20,800 Question Numbers 75 though 77 are based on the following:
Henderson Equipment Company has produced a pilot run of 50 units of a recently developed
71
. Assume again that Adrenal Company has sufficient capacity to produce 90,000 CADS each cylinder used in its finished products. The company expects to produce and sell 800 units. The
year. A customer in a foreign market wants to purchase 20,000 CADS. Import duties on the pilot run required 14.25 direct-labor hours for the 50 cylinders, averaging 0.285 direct-labor hours
CADS would be P1.70 per unit, and costs for permits and licenses would be P9,000. The only per cylinder. Henderson has experienced a significant learning curve on the direct-labor hours
selling costs that would be associated with the order would be P3.20 per unit shipping cost. needed to produce new cylinders. As cumulative output doubles, say from 25 to 50 units for
What is the break-even price on this order? example, the average labor time per unit declines by 20 percent. Past experience indicates that
A. P23.35 C. P22.15 learning tends to cease by the time 800 parts are produced. Hendersons manufacturing costs for
B. P28.65 D. P21.70 cylinders are as follows:
Direct labor P120.00 per hour
Variable overhead 100.00 per direct labor hour
72
. The company has 1,000 CADS on hand that have some deformities and are therefore Fixed overhead 166.00 per direct labor hour
considered to be seconds. Due to the deformities, it will be impossible to sell these units at Direct material 40.50 per unit
the normal price through regular distribution channels. What unit cost figure is relevant for
setting a minimum selling price? Henderson has received a quote of P75 per unit from the Leyte Machine Company for the
A. P16.80 C. P 4.70 additional 750 cylinders needed. Henderson frequently subcontracts this type of work and has
B. P18.00 D. P 1.20 always been satisfied with the quality of the units produced by Leyte. Recently, Henderson
Equipment Company has been operating at considerably less than full capacity.

30
Incremental Analysis

75
. How many direct-labor hours are expected to be used for the production of 800 cylinders A second alternative available to CLASP is to convert the special machinery to the standard model,
(including the pilot run)? which sells for P62,500. The additional identifiable costs for this conversion are as follows:
A. 93.4 hours C. 79.1 hours
B. 74.7 hours D. 67.6 hours Direct materials P2,850
Direct labor 3,300
76
. The production of 800 cylinders, including the pilot run, requires total incremental costs of: Total P6,150
A. P48,834 C. P68,452
B. P49,802 D. P52,948 A third alternative for CLASP is to sell the machine as is for a price of P52,000. However, the
potential buyer of the unmodified machine does not want it for 60 days. This buyer has offered a
77
. The effect on profit of producing 750 units instead of buying them from Leyte Machine P7,000 down payment, with the remainder due upon delivery.
Company a(an)?
A. Increase of P 8,470. C. Increase of P12,676. The following additional information is available regarding CLASPs operations:
B. Increase of P 7,052. D. Decrease of P22,560.
1. The sales commission rate on sales of standard models is 2 percent, while the rate on special
Questions 78 through 81 are based on the following: orders is 3 percent.
CLASP Industries received an order for a piece of special machinery from Tigok Company. Just as
CLASP completed the machine, Tigok declared backruptcy, defaulted on the order, and forfeited 2. Normal credit terms for sales of standard models are 2/10, net/30. This means that a
the 10 percent deposit paid on the selling price of P72,500. customer receives a 2 percent discount if payment is made within 10 days, and payment is
due no later than 30 days after billing. Most customers take the 2 percent discount. Credit
CLASPs manufacturing manager identified the costs already incurred in the production of the terms for a special order are negotiated with the customer.
special machinery for Tigok as follows:
3. The allocation rates for manufacturing overhead and fixed selling and administrative costs are
Direct material P16,600 as follows:
Direct labor 21,400 Manufacturing costs:
Manufacturing overhead: Variable 50% of direct-labor costs
Variable P10,700 Fixed 25% of direct-labor costs
Fixed 5,350 16,050 Fixed selling and administrative costs 10% of the total manufacturing costs
Fixed selling and administrative costs 5,405
Total P59,455 4. Normal time required for rework is one month.
78
Another company, Kay Corporation, will buy the special machinery if it is reworked to Kays . How much peso contribution would the sale to Kay Corporation add to CLASP before-tax
specifications. CLASP offered to sell the reworked machinery to Kay as a special order for profit?
P68,400. Kay agreed to pay the price when it takes delivery in two months. The additional A. P53,848 C. P55,900
identifiable costs to rework the machinery to Kays specifications are as follows: B. P55,948 D. P 9,300
79
Direct materials P 6,200 . How much peso contribution would the alternative of converting the special machinery to
Direct labor 4,200 standard model add to CLASPs before-tax profit?
Total P10,400 A. P52,200 C. P52,825

31
Incremental Analysis

B. P54,475 D. P 7,650 403 Direct labor hours 1 2 - 2


Machine hours 1 1 - 2
80
. If Kay makes CLASP a counteroffer, what is the lowest price CLASP should accept for the 405 Direct labor hours 2 2 2 1
reworked machinery from Kay? Machine hours 2 2 1 1
A. P10,400 C. P10,722
B. P12,500 D. P12,887 The sales department believes that the monthly demand for the next six months will be as follows:
81
. How much would the alternative of selling unmodified machinery to the potential buyer Product Monthly Unit Sales
contribute to CLASPs before-tax profit? 401 500
A. P50,440 C. P49,920 403 400
B. P 1,740 D. P49,400 405 1,000
Question Nos. 82 and 85 are based on the following: Inventory levels are satisfactory and need not be increased or decreased during the next six
Constraint Company manufactures and sells three products, which are manufactured in a factory months. Unit price and cost data that will be valid for the next six months are as follows:
with four departments. Both labor and machine time are applied to the products as they pass
through each department. The machines and labor skills required in each department are so
P R O D U C T S
specialized that neither machines nor labor can be switched from one department to another.
401 403 405
Constraint Companys management is planning its production schedule for the next few months. Unit costs:
The planning is complicated, because there are labor shortages in the community and some Direct material P 7 P 13 P 17
machines will be down several months for repairs. Direct labor
Department 1 12 6 12
Management has assembled the following information regarding available machine and labor time Department 2 21 14 14
by department and the machine hours and direct-labors required per unit of product. These data Department 3 24 -- 16
should be valid for the next six months. Department 4 9 18 9
Variable overhead 27 20 25
Fixed overhead 15 10 32
DEPARTMENT
Variable selling expenses 3 2 4
Monthly Capacity Available 1 2 3 4 Unit selling price P196 P123 P167
Norman machine capacity in MH 3,500 3,500 3,000 3,500
Capacity of machines being repaired 82
. Which department has capacity constraint in labor hours?
in machine hours ( 500) ( 400) ( 300) ( 200)
A. Department 1 C. Department 3
Available machine capacity in MH 3,000 3,100 2,700 3,300
B. Department 2 D. Department 4
Available direct labor hours (DLH) 3,700 4,500 2,750 2,600
83
. The total Machine Hours required by estimated monthly unit sales are:
Labor and Machine Specifications per
A. 10,600 C. 11,600
Unit of Product
B. 12,100 D. 13,500
Product Labor and Machine Time
401 Direct labor hours 2 3 3 1
Machine hours 1 1 2 2
32
Incremental Analysis

84
. The total number of labor hours as constraint for a month is: P40 per unit and the labor will be P72 per unit. Mr. Syjuco will have to spend P40,000 for a special
A. 50 C. 300 device which will be discarded when the job is done.
B. 750 D. No constraint
86
. What is the incremental cost of the special order of 5,000 units?
85
. In order to maximize its monthly profit, Constraint Company should produce: A. P600,000 C. P779,000
A. B. C. D. B. P421,000 D. P371,000
401 250 250 500 500
87
403 0 400 400 0 . What is the full cost of the special order?
405 1,000 1,000 625 625 A. P779,000 C. P421,000
B. P492,400 D. P651,000
Question Nos. 86 through 89 are based on the following; 88
Arnold Syjuco operates a small machine shops. He manufactures one standard product available . The amount of opportunity cost of taking the special order is:
from many other similar businesses and he also manufactures products to customer order. Hi A. P183,000 C. P250,000
accountant prepared the annual income statement shown below: B. P 71,000 D. P124,600
89
Custom Sales Standard Sales Total . What is the effect on the overall profit if the special order is accepted?
A. P450,000 C. P( 25,000)
Sales P1,000,000 P500,000 P1,500,000
B. P( 85,000) D. P 29,000
Material P 200,000 P160,000 P 360,000
Labor 400,000 180,000 580,000
Question Nos. 90 through 94 are based on the following:
Depreciation 126,000 72,000 198,000
The Verbatim Corporation, which produces and sells to wholesalers a highly successful line of
Power 14,000 8,000 22,000
summer lotions and insect repellents, has decided to diversify in order to stabilize sales over the
Rent 120,000 20,000 140,000
year. A natural area for the company to consider is the production of special lotion and cream to
Heat and light 12,000 2,000 14,000
prevent dry and chapped skin.
Other 8,000 18,000 26,000
Total P 880,000 P460,000 P1,340,000
After considerable research, a special product line has been developed. However, because of the
Income P 120,000 P 40,000 P 160,000
conservatism of the company management, Verbatims president has decided to introduce only
one of the new products for this coming rainy season. If the product is a success, further
The depreciation charges are for machines used in the respective product lines. The power charge expansion will be initiated in future years.
is apportioned on the estimate of power consumed. The rent is for the building space which has
been leased for 10 years at P140,000 per year. The rent and heat and light are apportioned to the The product selected (called Chaps) is a lip balm that will be sold in a lipstick-type tube. The
product lines based on amount of floor space occupied. All other costs are current expenses product will be sold to wholesalers in boxes of 24 tubes for P800 per box. Because of available
identified with the product line incurring them. capacity, no additional fixed charges will be incurred to produce the product. However, a
P10,000,000 fixed charge will be absorbed by the new product to allocate a fair share of the
A valued custom parts customer has asked Mr. Syjuco to manufacture 5,000 special units for him. companys present fixed costs to it.
Mr. Syjuco is working at capacity and would have to give up some other business to take this
business. He cannot renege on custom orders already agreed to but he could reduce the output of Using the estimated sales and production of 100,000 boxes of Chaps as the standard volume, the
his standard product by about one-half for one year while producing the specially requested accounting department has developed the following costs:
custom part. The customer is willing to pay P140 for each part. The material cost will be about

33
Incremental Analysis

Direct labor P200 per box Unit manufacturing costs:


Direct materials 300 per box Direct materials P1,000
Total overhead 150 per box Direct labor 1,500
Total P650 per box Variable overhead 500
Fixed overhead 1,200 P4,200
Verbatim has approached a cosmetics manufacturer to discuss the possibility of purchasing the Unit marketing costs:
tubes for Chaps. The purchase price of the empty tubes from the cosmetics manufacturer would Variable 500
be P90 per 24 tubes. If the Verbatim Corporation accepts the purchase proposal, it is estimated Fixed 1,400 1,900
that direct labor and variable overhead costs would be reduced by 10% and direct material costs Total unit costs P6,100
would be reduced by 20%.
Unless otherwise stated, assume there is no connection between the situations described in the
90
. What is the variable overhead rate per box of Chaps? questions; each is to be treated independently. Unless otherwise stated, a regular selling price of
A. P100 C. P 50 P7,400 per unit should be assumed. Ignore income taxes and other costs that are not mentioned
B. P150 D. P200 in the cost schedule or in a question itself.
95
. What is the monthly breakeven units for Medical Supply Company?
91
. What is the material cost per box of Chaps saved by purchasing them? A. 2,000 C. 1,950
A. P300 C. P 60 B. 2,689 D. 2,614
B. P240 D. P 30
96
. Market research estimates that volume could be increased to 3,500 units, which is well within
92
. How much would it cost Verbatim to produce the tubes per box? hoist production capacity limitations, if the price were ct from P7,400 to P6,500 per unit.
A. P 60 C. P 90 Assuming the cost behavior patterns implied by the data in the cost schedule is correct, would
B. P 85 D. P120 you recommend this action be taken?
A. Yes, because the profit will increase by P1,500,000.
93
. How much would Verbatim incur by making 125,000 boxes, assuming that additional B. Yes, because the profit will increase by P 200,000.
equipment, at an annual rental of P1,000,000, must be acquired to produce this volume? C. No, because the profit will decrease by P1,200,000.
A. P10,625,000 C. P11,250,000 D. No, because the profit will decrease by P2,400,000.
B. P11,625,000 D. P12,500,000
97
. On March 1, a contract offer is made to Medical Supply Company by the Veterans Hospital to
94
. Referring to Question No. 93, what is the impact on its profit if Verbatim were to buy 125,000 supply 500 units for delivery by March 31. Because of an unusually large number of rush
boxes? orders form their regular customers. Medical Supply plans to produce 4,000 units during
A. Additional profit of P1,000,000. C. Additional profit of P375,000. March, which will use all available capacity. If the Veterans Hospitals order is accepted, 500
B. Additional profit of P1,250,000. D. Decrease in profit of P625,000. units normally sold to regular customers would be lost to a competitor. The contract given by
the hospital would reimburse the Veterans Hospitals share of March manufacturing costs,
Question Nos. 95 through 101 are based on the following: plus pay a fixed fee (profit) of P500,000. (There would be no variable marketing costs incurred
Medical Supply Company produced hydraulic hoists that were used by hospitals to move on the hospitals unit.) What impact would accepting the Veterans Hospital contract have on
bedridden patients. The costs of manufacturing and marketing hydraulic hoists at the companys March income?
normal volume of 3,000 units per month are show below: A. P 1,100,000 C. P(1,350,000)
B. P( 850,000) D. P 500,000

34
Incremental Analysis

98
. Medical Supply Company has an opportunity to enter a foreign market in which price Question Nos. 102 and 103 are based on the following:
competition is keen. An attraction of the foreign market is that demand there is greatest when Marcus Fibers, Inc., specializes in the manufacturing of synthetic fibers that the company uses in
demand in the domestic market is quite low; thus idle production facilities could be used many products such as blankets, coats, and uniforms for police and firefighters. Marcus has been
without affecting domestic business. in business since 1975 and has been profitable every year since 1983. The company uses a
An order for 1,000 units is being sought at a below-normal price in order to enter this market. standard cost system and applies overhead on the basis of direct labor hours.
Shipping costs for this order will amount to P750 per unit, while total costs of obtaining the
contract (marketing costs) will be P40,000. No other variable marketing costs would be Marcus has recently received a request to bid on the manufacture of 800,000 blankets scheduled
required on this order. Domestic business would be unaffected by this order. What is the for delivery to several military bases. The bid must be started at full cost per unit plus a return on
minimum unit price should Medical Supply Company consider for this order of 1,000 units? full cost of no more than 9 percent after income taxes. Full cost has been defined as including all
A. P3,750 C. P3,790 variable costs of manufacturing the product, a reasonable amount of fixed overhead, and
B. P3,000 D. P4,290 reasonable incremental administrative costs associated with the manufacture and sale of the
product. The contractor has indicated that bids in excess of P25 per blankets are not likely to be
99
. An inventory of 230 units of an obsolete model of the hoist remains in the stockroom. These considered.
must be sold through regular channels at reduced prices, or the inventory will soon be
valueless. What is the minimum price that would be acceptable in selling these units? In order to prepare the bid for the 800,000 blankets, Andrea Lighter, cost accountant, has gathered
A. P3,500 C. P3,000 the following information about the cost associated with the production of the blankets.
B. P4,200 D. P 500
Direct material P 1.50 per pound of fibers
100
. A proposal is received from an outside contractor who will make and ship 1,000 hydraulic hoist Direct labor P 7.00 per hour
units per month directly to Medical Supplys customers as orders are received from Medical Direct machine costs* P10,00 per blanket
Supplys sales staff. Medical Supplys fixed marketing costs would be unaffected, but its Variable overhead P 3.00 per direct labor hour
variable marketing costs would be cut by 20 percent for these 1,000 units produced by the Fixed overhead P 8.00 per direct labor hour
contractor. Medical Supplys plant would operate at two thirds of its normal level, and total Incremental administrative costs P2,500 per 1,000 blankets
allocated fixed manufacturing costs for these 1,000 units would be cut by 30 percent. What in- Special fee** P 0.50 per blanket
house unit cost should be used to compare with the quotation received from the supplier? Material usage 6 pounds per blanket
A. P 3,760 C. P 4,240 Production rate 4 blankets per DLH
B. P 3,000 D. P 3,460 Effective tax rate 40%
101 102
. Assume the same facts as in requirement No. 101 except that the idle facilities would be used . The minimum price per blanket that Marcus Fibers, Inc., could bid without reducing the
to produce 800 modified hydraulic hoists per month for us in hospital operating rooms. These companys net income is
modified hoists could be sold for P9,000 each, while the costs of production would be P5,500 A. P24.00 C. P50.25
per unit variable manufacturing expense. Variable marketing costs would be P1,000 per unit. B. P21.50 D. P40.25
Fixed marketing and manufacturing costs would be unchanged whether the original 3,000
103
regular units hoists were manufactured or the mix of 2,000 regular hoists plus 800 modified . Using the full-cost criteria and the maximum allowable return specified, Marcus Fibers bid
hoists were produced. What is the maximum purchase price per unit that Medical Supply price per blanket would be:
should be willing to pay the outside contractor? A. P24.00 C. P26.00
A. P 5,100 C. P 5,500 B. P29.90 D. P27.90
B. P 3,100 D. P 5,600

35
Incremental Analysis

ANSWER EXPLANATIONS

36
1
. Answer: C
Cost of alternative selected P800,000
Cost of alternative rejected 650,000
Incremental cost P150,000
2
. Answer: A
The company needs to purchase 55,000 units earlier than their scheduled 5,000-unit monthly purchase. Hence, the
average investment for the inventory is (55,000 x P60 2) or P1,650,000. The opportunity cost is P132,000 or
(P1,650,000 x 0.08).
3
. Answer: A
Additional revenue after rework (24,000(12 4) P192,000
Less Additional cost (24,000 x 2) 48,000
Additional profit P144,000
4
. Answer: B
The only relevant out-of pocket cost is the variable selling expense which is P40. The sale thru the regular channels
involves an opportunity cost of P140.
Variable selling expense (40% x 100) 40
Opportunity cost 140
Total 180
5
. Answer: C
Regular variable cost P8.00
Overtime premium 1.50
Relevant cost per unit P9.50
6
. Answer: B
Full cost 50.00
Fixed overhead (180,000/9,000) 20.00
Relevant unit cost 30.00
7
. Answer: C
Cost of 1,000 kg at latest price (1,000 x 8.70) 8,700
Add excess price include on the remaining 4,000 kg. 4,000 x (8.70 8.30) 1,600
Relevant cost 10,300
8
. Answer: B
Direct materials (2,000 @ 10) 20,000
Direct labor (2,000 @ 12) 24,000
Variable overhead (2,000 @ 6) 12,000
Increase in variable cost due to overtime (2,000 @ 14) 28,000
Incremental cost 84,000
9
. Answer: B
Variable costs P56,250
Additional fixed costs 13,750
Minimum bid price P70,000
10
. Answer: B
Direct material (360,000 24,000) P15.00
Direct labor (540,000 24,000) 22.50
Variable selling expenses (84,000 24,000) 3.50
Total P41.00
Add Profit per unit (22,500 1,500) 15.00
Selling price P56.00
11
. Answer: B
Relevant cost to make and sell:
Direct materials 39
Direct labor 6
Variable OH 8
Reduced selling expenses (30 x 0.06) 18
Addl fixed cost (20,000 5,000) 4
Minimum selling price 75
12
. Answer: B
The company has no existing capacity. The minimum selling price for this special sales should equal the regular selling
price plus additional expenses.
Regular selling price P18
Additional expenses 1
Minimum selling price P19
13
. Answer: A
Direct materials 20.00
Direct labor 15.00
Variable overhead 12.00
Variable shipping and handling 3.00
Lost contribution margin LB46 (10,000 1,000) 10.00
Minimum price 60.00
The lost contribution margin on regular sale is relevant because the company is operating at capacity. In a special sale
wherein the company has to give up some of its regular units, the relevant costs consist of incremental costs plus any
opportunity costs.
14
. Answer: D
Direct materials 4.50
Direct labor 10.00
Variable overhead 3.00
Variable selling expense 1.00
Additional profit (40,000/5,000) 8.00
Required selling price 26.50
15
. Answer: C
The maximum number of units in regular sales that Benjing could afford to lose equals the quantity that provides regular
contribution margin that matches the contribution margin provided by special sale.
Contribution margin from special sale 1,000 (14 8) 6,000
Divided by regularCM (20 8) 12
Maximum Number of units 500

To illustrate the solution:


Contribution margin from special sale 6,000
Less Decrease in regular sales contribution margin (500 x 12) 6,000
Effect on profit NIL
16
. Answer: B
The maximum decrease in regular sale = Contribution margin from special sale/Unit contribution margin on regular
sale
(400 x 0.20) (2.00 -1.50) = 160
17
. Answer: A
Total contribution margin from special sale(15,000 x P5.50) P82,500
Less Additional fixed costs 30,000
Profit from special sale P52,500
Less Decrease in contribution margin on regular
Sale 2,000(P39 P22.50) 33,000
Additional profit P19,500
Please refer to Solution for Number regarding details of contribution margin per unit.
18
. Answer: C
Selling price P6.00
Relevant cost per unit:
Regular cost per unit P7.50
Less: Commission P0.75
Fixed overhead (P3 x 2/3) 2.00 (2.75)
Net amount P4.75
Incremental fixed cost (P150 /300) 0.50 5.25
Advantage per unit, Buy P0.75
Number of units 300
Increase in profit P 225
19
. Answer: D
Additional profit: 3,000 x (70 50) = 60,000
20
. Answer: A
Special price 32
Relevant cost:
Direct materials 4
Direct labor 12
Variable overhead 6 22
Unit contribution margin 10
Units ordered 2,000
Additional profit 20,000
21
. Answer: A
Sales 60,000
Less: Variable production cost (1,500 x 30) 45,000
Additional Fixed cost 5,000
Labeling cost (1,500 x 2.50) 3,750 53,750
Profit 6.250
22
. Answer: B
The minimum selling price should equal the relevant cost to produce and sell a unit of product.
Direct materials P50
Direct labor 20
Variable overhead (P10 x 0.2) 2
Selling expense (P15 x 0.4) 6
Minimum selling price P78
23
. Answer: C
The company has no excess capacity to be devoted to the production of additional units for special sale. In a special
sale decision where there is no excess capacity, the minimum selling price must be equal to the market price less any
avoidable expenses.
Selling price P100
Less Avoidable selling expense (P15 x 0.4) 6
Minimum selling price P 94
24
. Answer: D
The book value of the old equipment is a sunk cost and therefore not a relevant one. Also, the related cost on
outstanding note are irrelevant. They are not affected by a decision.
25
. Answer: D
Relevant Costs
BetaZetaDirect materials 4.00 80.00Direct labor10.00 47.00Factory overhead 40% 16.00 8.00Relevant Unit
costP30.00135.00
26
. Answer: C
Direct material 2.00
Direct labor 2.40
Variable overhead 1.60
Avoidable marketing cost (0.7 x 2.50) 1.75
Relevant cost Make 7.75
The maximum purchase price, if ever the company has to decide buying the product, is P6.75. Any amount higher than
P6.75 will necessarily increase the unit cost of the product.
27
. Answer: B
Direct materials 64,000
Direct labor 16,000
Variable overhead 8,000
Avoidable fixed overhead 6,000
Total relevant cost to make 94,000
28
. Answer: D
Direct materials 64,000
Direct labor 16,000
Variable overhead 8,000
Additional contribution margin 16,000
Total relevant cost to make 104,000
29
. Answer: B
Variable cost to make parts (30,000 x 12) 360,000
Cost buy (30,000 x 15) 450,000
Cost savings Make decision 90,000
30
. Answer: A
Direct materials 80,000
Direct labor 13,000
Variable overhead 40,000
Avoidable fixed overhead 4,000
Relevant cost make 137,000
Purchase price 145,000
Advantage Make 8,000
31
. Answer: A
Direct materials 64,000
Direct labor 16,000
Variable overhead 8,000
Total variable cost 88,000
Less Purchase cost 104,000
Avoidable fixed cost 16,000
Add unavoidable FC 5,000
Total fixed overhead 21,000
32
. Answer: B
Purchase cost (2,000 x P15) P30,000
Relevant cost to make:
Variable cost (2,000 x P16) 8,000 P24,000
Avoidable fixed cost (8,000 x 0.25) 2,000 26,000
Additional cost Buy (Decrease in profit) P 4,000

Alternative computation for relevant cost to make:


Total cost (2,000 x P16) P32,000
Less unavoidable fixed cost (8,000 x 0.75) 6,000
Relevant cost to make P26,000
33
. Answer: C
Cost of purchase (2,000 x P15) P30,000
Relevant cost make:
Variable cost (2,000 x P16) P8,000 P24,000
Avoidable fixed cost (P8,000 x 0.25) 2,000
Opportunity cost rent 6,000 32,000
Cost savings Buy (increase in profit) P( 2,000)
34
. Answer: C
Relevant costs to make
Direct materials P 90,000
Direct labor 120,000
Variable overhead 15,000
Supervisors salary 30,000
Opportunity costs, rent 65,000
Total 320,000
Relevant cost to buy (15,000 x P20) 300.000
Advantage - Buy P 20,000
If the company would purchase the units, it would save P20,000.
35
. Answer: B
Cost of ink cartridges (5,000 x P15) P75,000
Less: Relevant cost to produce (5,000 x P8) 40,000
Additional cost if ink cartridges are purchased P35,000
36
. Answer: B
Direct material (20,000 @ 6) 120,000
Direct labor (20,000 @30) 600,000
Variable overhead (20,000 @ 120 240,000
Avoidable fixed cost (20,000 @ 9) 180,000
Total relevant costs - Make 1,140,000
Purchase cost (20,000 @ 60) 1,200,000
Add net savings 25,000
Total 1,225,000
Less: Cost to make 1,140,000
Opportunity cost 85,000
37
. Answer: A
Purchase price 15,000
Handling cost (20% x P15,000) 3,000
Total 18,000
Cost to make (21,200 8,000)* 13,200
Increase in unit cost if goods are purchased 4,800

*Fixed OH (12,000 x 2 3) = 8,000


38
. Answer: A
Cost to make:
Direct materials P10,000
Direct labor 5,000
Variable overhead 5,000
Avoidable fixed OH (20% x 30,000) 6,000
Relevant cost P26,000
Purchase costs (1,000 @ 30) 30,000
Decrease in profit in profit P 4,000
39
. Answer: B
Relevant costs to make per unit:
Direct materials 6.00
Direct labor 4.00
Variable overhead 1.00
Relevant cost to make 11.00
Purchase price per unit 12.50
Increase in per unit cost if purchased 1.50
40
. Answer: A
Direct materials 10,000
Direct labor 5,000
Variable overhead 5,000
Avoidable fixed overhead (30,000 x 0.2) 6,000
Total relevant cost 26,000
Purchase cost 30,000
Additional cost if purchased 4,000
41
. Answer: C
Direct materials 3.00
Direct labor 15.00
Variable overhead 6.00
Avoidable fixed cost 3.00
Total per unit 27.00
Number of unit x10,000
Total 270,000
Add savings from the manufacture of other product 45,000
Total relevant cost make 315,000
Total purchase cost (10,000 x 30) 300,000
Advantage Buy 15,000
42
. Answer: D
Though the problem deals with transfer of goods from one division to another division, the solution focuses on make on
buy decision approach.
Purchase price, outside supplier 1.25
Variable cost to make (10,000 10,000) 1.00
Additional unit cost to the company 0.25
Units to be purchased 10,000
Decrease in Danas profit if goods are purchased 2,500
43
. Answer: C
Total purchase cost (5,000 x 7.75) 38,750
Less Relevant cost to make
Direct materials @ 2.5 12,500
Direct labor @ 3.5 17,500
Variable overhead @ 1.5 7,500
Avoidable fixed cost @ 0.5 2,500
Opportunity cost 6,000 46,000
Net saving purchase (7,250)
44
. Answer: D
The solution is made in equation form, using y = a + bx for 2 alternatives:
Let x = indifference point in units
Make: y = 150,000 + 11x
Buy: y = 60,000 + 12.875x
150,000 + 11x = 60,000 + 12.875x
1.875x = 60,000
x = 48,000
45
. Answer: C
Direct materials 80,000
Direct labor 13,000
Variable overhead 40,000
Avoidable fixed overhead 20,000
Total relevant cost 153,000
46
. Answer: B
SodaLemomadeSelling price6.005.00Variable cost 1.501.00Contribution margin4.504.00Processing
hours34CM/Hr1.501.00For the Lemonade to be as profitable as Soda, its contribution margin per hour should be P1.50.

Therefore the required selling price for Lemonade is P7, calculated as:
Contribution margin per unit (4 hours x P1.50) P6.00
Variable cost per unit 1.00
Selling price P7.00
47
. Answer: C
Product B has a greater contribution margin per unit (P15 - P12 = P3) than Product A (P12 - P10 = P2). The company
should produce the maximum units it can sell of Product B (250,000) and use the rest of the machine hour capacity to
produce 100,000 units of Product A.
48
. Answer: D
Production order: Y, X
Product X: 60 6 = 10
Product Y: 42 3 = 14
Total capacity MH 42,000
Machine hours devoted to Product Y (8,000 x 3) 24,000
Hours available to X 18,000
Production of X: 18,000 6 = 3,000
49
. Answer: B
Production order:
BlenderElectric MixerPurchase price 20 38Variable cost to make: Direct materials 6 11 Direct materials 4 9 Overhead
*(16 10) @ 6 12 Total( 16) (32)Additional cost if purchased 4 6Additional cost per hour (Blender, 1 hr; Mixer 2 hours)
4
3
Since it will cost Mary P4 per hour to buy Blender and only P3 if Electric Mixer is purchased, it will produce all of
Blenders requirement and just purchase units of electric mixer that cannot be accommodated by the remaining capacity.

Product:
Blender 20,000
Electric Mixer [50,000 (20,000 @ 1)] 2 15,000
Purchase:
Electric Mixer (28,000 15,000) 13,000
50
. Answer: A
CM Product A 36/2 x 1,000 18,000
CM Product B 45/3 x 1,000 15,000
Difference in contribution margin 3,000
51
. Answer: A

Based on DLH
ProductsUCMDLH/unitCM/DLHPriorityA91.09.01STB101.56.672ndC82.04.003rd
Based on MH
ProductsUCMMH/unitCM/MHPriorityA94.52.03rdB1025.01stC82.53.22nd
52
. Answer: D
PlasticMetalRC make 11.00 13.00RC Buy 15.50 17.50Additional Cost-Buy 4.50 4.50Hours required/unit 3
4.5Additional cost /hr. 1.50 1.0Priority 1st 2ndCapacity (machine hours) 48,000MH used - Plastic (7,000 x 3)
21,000Available MH to Metal27,000MH used - Metal (6,000 x 4.50) (27,000)Purchase of Metal (11,000 6,000) 5,000
53
. Answer: A
HomeDeluxeProSelling price586580Direct materials(16)(20)(19)Direct labor(10)(15)(20)Variable overhead( 8)(12)
(16)CM/unit241825Processing hour(s) 1 1.5 2CM/DLH2412 12.50Profitability rank1st3rd2nd
54
. Answer: B
Unit contribution margin:
Product A P12 P10 P2
Product B P15 P12 P3

Contribution margin per hour:


Product A P2 0.8 P2.50
Product B P3 1.25 P2.40

Total capacity in hours 350,000


Less hours used by Product A 200,000 x 0.8 (160,000)
Available hours for production of Product B 190,000
Less hours by Product B 152,000 x 1.25 (190.000)
Number of units to be produced:
Product A 200,000
Product B 152,000

Product A has higher contribution margin per hour. The company should produce the maximum units it can sell of
Product A and use the rest of the machine hour capacity to produce units of Product A in order to maximize its profit.
55
. Answer: B
CM per hour:
Product X: 50/5 10
Product Y: 64/8 8
The 20,000 hours (0.8 x 25,000) will be devoted to the production of X.
Total contribution margin: (20,000 x 10) + (5,000 x 8) 240,000
56
. Answer: A
Selling price per unit silver polish P40
Less variable costs:
Grit 337 (P20 4) P 5
Ingredients, direct labor and variable OH 25
Variable selling costs 3 33
Contribution margin per unit P 7

Minimum number of jars of silver polish to be produced:


Avoidable fixed costs Contribution margin per jar P56,000 P7 8,000

The solution used the selling price of P20 as cost of Grit337 because there was unlimited demand for the cleaning
powder. If, however, the demand for the cleaning powder is limited, the recommended solution would use P16 as the
cost of Grit 337.
57
. Answer: D
Increase in selling price 116 90 26
Additional processing cost 18
Addition profit per unit 8
58
. Answer: C
Selling price after further processing P8.75
Selling price if not processed further 7.20
Additional sales per unit 1.55
Number of units 25,000
Additional total sales P38,750
Less additional processing costs 33,750
Increase in profit if the product is processed P 5,000
Because further processing will provide more profit per unit, the company should process further.
59
. Answer: D
Additional sales (350,000 250,000) P100,000
Additional costs (75,000 + 15,000) 90,000
Additional profit P 10,000
60
. Answer: A
XYAdditional sales value1034Additional processing costs1530Incremental (decremental) profit per unit(5) 4If Product Y
is processed further, profit will increase by P16,000 (4,000 x 4).
61
. Answer: A
Additional Sales Price (65 50) 15.00
Additional Cost (30 x 40%) 12.00
Additional profit 3.00
62
. Answer: C
Product to be processed further:Prod MProd NFinal selling price3123Selling price at split-off point2519Increase in selling
price64Units15,00030,000Total increase in sales90,000120,000Additional processing costs100,000110,000Increase
(decrease) in profit(10,000)10,000
63
. Answer: C
Revenues P104,000
Avoidable costs:
Cost of goods sold P 64,000
Avoidable expenses (P6,000 + P8,000) 14,000 78,000
Segment margin P 26,000
A segment is a potential candidate for elimination if its revenues are less than its avoidable costs. This is not the case
for this segment. The company will lose P26,000 of income if this segment is eliminated.
64
. Answer: A
Avoidable fixed cost (benefit) 210,000
Lost contribution margin 240,000
Decrease in profit 30,000
65
. Answer: D
The question did not require any computation. If Mina Co. drops the Gold Ore, it will lose the segment margin of
P1,200,000, a decrease in Mina Co.s income. The amount of direct fixed expenses that would be eliminated were
previously deducted from contribution margin, and therefore, not considered in the determination of the effect on
income.
66
. Answer: B
Avoidable common expenses (45,000 20,000) P 25,000
Segment margin lost 32,000
Decrease in profit P (7,000)
67
. Answer: A
Avoidable fixed expenses:
Manufacturing (150,000 105,000) 2 90,000
Selling (30,000 x 0.10 x 2) 6,000
Start up cost (additional fixed expense ( 8,000)
Net avoidable costs 88,000
Indifference point 88,000 (22-14) 11,000 units
At 11,000 unit level (2 months), the contribution margin equals the avoidable costs.
68
. Answer: D
Total Savings 5 year (125,000 100,000 ) 5 125,000
Less:
Additional depreciation (90,000 50,000) (40,000)
Loss on sale of old machine (5,000 50,000) (45,000)
Increase in profit 40,000
69
. Answer: A
Lease arrangement:
Rental income (5 years) 48,000
Cost of repairs, insurance and property taxes 10,000
Net income 38,750

Sale arrangement:
Net proceeds (25,000 x 0.95) 23,750
Differential income lease 15,000
70
. Answer: A
Additional contribution (60,000 x 0.25 x 14) 210,000
Additional fixed selling costs 80,000
Additional profit 130,000

Selling price 32.00


Variable expenses:
Materials 10.00
Direct labor 4.50
Variable overhead 2.30
Variable selling costs 1.20 18.00
Unit contribution margin 14.00
71
. Answer: C
Direct materials 10.00
Direct labor 4.50
Variable OH 2.30
Variable selling cost 3.20
Import duties 1.70
Permits and licenses (9,000 20,000) 0.45
Minimum selling price 22.15

Import duties are assumed to be paid by Adrenal Company because of the nature of the sale.
72
. Answer: D
The relevant cost in selling the units on hand (inferior quality) is P1.20, the variable selling costs the production costs,
though variable, are considered irrelevant because they are historical (sunk) costs.
73
. Answer: C
Avoidable fixed costs:
Manufacturing (0.40 x 50,000) 20,000
Selling (35,000 x 0.20) 7,000
Total 27,000

Contribution margin if the company has to operate (60,000 6 x 0.30 x 14) 42,000
Disadvantage, closing the plant (15,000)
74
. Answer: A
Direct materials 10.00
Direct labor 4.50
Variable overhead 2.30
Avoidable fixed overhead (0.75 x 5) 3.75
Avoidable variable expense (1.20 x 1 3) 0.40
Relevant cost Make 20.95
75
. Answer: A
Batch (each 50 units)Cum Ave. Hrs114.25211.4049.1287.296165.8368
Total Hours required: 16 x 5.8368 = 93.4
76
. Answer: D
Materials (800 x 40.50) 32,400
Direct labor (93.40 x 120) 11,208
Variable OH (93.40 x 100) 9,340
Total 52,948
77
. Answer: A
Production cost 750 units:
Materials (750 x 40.00) 30,375
Direct labor (93.40 14.25) 120 9,498
VOH (93.40 14.25) 100 7,915
Total 47,780
Purchase cost (750 x 75)
Advantage make 56,250
8,470
78
. Answer: A
Sales price to Kay Corp. 68,400
Rework costs:
Direct materials 6,200
Direct labor 4,200
Variable OH (4200 x 50%) 2,100 (12,500)
Commission (68,400 x 0.03) ( 2,052)
Before tax peso contribution 53.848
79
. Answer: A
Regular price 62,500
Deduct:
2% commission (62,500 x 0.02) 1,250
Sales discount (62,500 x 0.02) 1,250 2,500
Net price 60,000
Less additional conversion costs:
Direct materials 2,850
Direct labor 3,300
Variable OH - 50% 1,650 7,800
Net before tax contribution 52,200
80
. Answer: D
Cost of rework 6,200
Direct labor 4,200
Variable OH (4,200 x 0.50) 2,100
Total 12,500
Commission [0.03 (12,500 0.97)] 387
Total 12,887
81
. Answer: A
Sales price 52,000
Less: Commission (52,200 x 0.03) 1,560
Net contribution 50,440
82
. Answer: C
Department 3 has constraint in labor hours of 750.
Dept. 1Dept. 2Dept. 3Dept. 4Available DLH3,7004,5002,7502,600DLH required4011,0001,5001,500 500402 400 800 --
8004032,0002,0002,0001,000 Total3,4004,3003,5002,300Excess (Constraint) 300 200( 750) 300
83
. Answer: A
The available machine hours are sufficient to produce the estimated monthly sales. The schedule for monthly
production should consider maximizing the use of available direct labor hours in Department 3 because it is the only one
with constraint.
Dept. 1Dept. 2Dept. 3Dept. 4Available MH3,0003,1002,7003,300MH required401 500 5001,0001,000402 400 400 --
8004032,0002,000 1,0001,000 Total2,9002,9002,0002,800Excess (Constraint) 100 200 700 500
Total machine hours required by monthly unit sales: (2,900 + 2,900 + 2,000 + 2,800) 10,600
84
. Answer: B
The table showing the comparison of available hours and required hours to produce all the required units in number 82
indicated that Department 3 is short by 750 hours. Any excess direct labor hours in the other departments cannot be
switched to Department 3.
85
. Answer: B
The production plan that will maximize monthly profit should be based on the profitability of the three products in terms
of the use of direct labor hours in Department 3.
P R O D U C T S401403405Selling price per unitP196P123P167Variable unit costsDirect material71317Direct
labor663851Variable overhead272025Selling expenses324 Total variable cost1037397Unit contribution marginP 93P
50P 70No. of DLH required Dept 33-2Contribution margin per DLHP 31-P 35Based on the above schedule, Product
405 is more profitable per hour than Product 401s and, therefore, all of the units required for Product 405 should be
produced. Product 403 would not use any direct labor hours in Department 3 and so all of the required units for Product
403 can be produced.

Available direct labor hours Department 3 2,750

Hours used by Product 405 1,000 x 2 2,000


Available hours for Product 401 750

Production units Product 401 250 x 3 750

Production:
Product 401 250
Product 403 400
Product 405 1,000

Alternative Solution:

Since Product 401 is the less profitable per DLH, Product 403 and 405 will be produced in full and Product 401 will be
partially produced.

Total required units, Product 401 500


Equivalent units based on constraint 750 3 250
Production of Product 401 250

Alternative question: What is the maximum monthly contribution margin that Constraint Company can earn?
Product 401 250 @ P93 P 23,250
Product 403 400 @ P50 20,000
Product 405 1,000 @ P70 70,000
Total contribution margin P113,250
86
. Answer: B
Costs incurred to make the order:
Material (5,000 x 40) P200,000
Labor (5,000 x 72) 360,000
Incremental fixed cost (special device) 40,000
Costs to be incurred P600,000

Decrease in costs for standard products:


Material (0.5 x 160,000) P 80,000
Labor (0.5 x P180,000) 90,000
Other (0.5 x P18,000 9,000
Decrease in costs P179,000
Net incremental costs P421,000

The amounts for depreciation, rent, and heat and light are assumed to be not affected by the special order. There is no
information provided as to how power cost was exactly incurred.
87
. Answer: D
Costs to be incurred for special order P600,000
Fixed costs:
Depreciation (0.5 x 72,000) P36,000
Power (0.5 x 8,000) 4,000
Rent (0.5 x 20,000) 10,000
Heat and Light (0.5 x 2,000) 1,000 51,000
Total cost P651,000
The amount of fixed costs allocated to special order would be the costs that should have been assigned to the
standard sales that would be cancelled.
88
. Answer: B
Decrease in sales of standard products0.50 x 500,000 P250,000
Less variable costs:
Material (160,000 x 0.5) P80,000
Labor (180,000 x 0.5) 90,000
Other (18,000 x 0.5) 9,000 179,000
Opportunity costs P 71,000
89
. Answer: D
Special sales (5,000 x 140) P700,000
Variable costs 600,000
Contribution margin from special sale 100,000
Less opportunity costs 71,000
Increase in profit P 29,000
90
. Answer: C
Total overhead rate per box P150
Less fixed overhead allocated per boxP10,000,000 100,000 boxes 100
Variable overhead rate per box P 50
91
. Answer: C
The cost of materials saved by a decision of purchasing the tubes: is P300 x 0.20 = P 60
92
. Answer: B
The relevant cost to make the tubes by Verbatim should equal the amount of cost savings as follows:
Savings on materials 0.2 x P300 P 60
Labor 0.1 x P200 20
Overhead 0.1 x P 50 5
Total savings (relevant cost) P 85
The maximum amount that Verbatim is willing to pay per box of 24 tubes must be P85.
93
. Answer: B
Cost of making 125,000 boxes:
Variable costs 125,000 x 85 10,625,000
Additional fixed costs 1,000,000
Total 11,625,000
94
. Answer: C
Total purchase cost 125,000 x 900 11,250,000
Total cost to make 125,000 x 85 11,625,000
Savings if purchased 375,000
95
. Answer: A
Fixed costs:
Manufacturing 3,000 x 1,200 P3,600,000
Marketing 3,000 x 1,400 4,200,000
Total P7,800,000

Selling Price P 7,400


Less Variable costs:
Direct materials P1,000
Direct labor 1,500
Variable overhead 500
Marketing costs 500
Total 3,500
Unit contribution margin P 3,900

Breakeven units 7,800,000 3,900 2,000 units


96
. Answer: C
In as much that there would be no change in the amount of fixed costs, the recommended solution was made by just
comparing the amounts of contribution margin based on the revised data and the original information:

Contribution margin based on new estimates 3,500 x (6,500 3,500) 10,500,000


Contribution margin based on current estimates
Decrease 3,000 x (7,400 3,500) 11,700,000
Decrease in profit ( 1,200,000)

Alternative Solution:
Total contribution margin 3,000 x (7,400 3,500) 11,700,000
Less Fixed costs 7,800,000
Current profit 3,900,000

Total contribution margin at reduced price 3,500 x (6,500 3,500) 10,500,000


Less Fixed costs 7,800,000
Revised profit 2,700,000
Current profit 3,900,000
Decrease in profit ( 1,200,000)
97
. Answer: B
Fixed fee P 500,000
Fixed overhead reimbursement 500 x 1,200 600,000
Total 1,100,000
Less lost contribution margin on regular customers (500 x 3,900) 1.950,000
Decrease in profit P( 850,000)
The reimbursement for fixed overhead is an income for Medical Hospital Company because the special order does not
entail additional fixed overhead.
98
. Answer: C
Direct materials 1,000
Direct labor 1,500
Variable overhead 500
Shipping cost 750
Cost of obtaining the order 40,000 1,000 40
Minimum selling price 3,790
99
. Answer: D
All the production costs, both variable and fixed, are no longer relevant because they are sunk costs. To be relevant to
a decision, the cost must be both valid and relevant. Therefore, the only relevant cost is the variable marketing cost,
because if the units will be sold through regular channel, P500 will be incurred.
100
. Answer: D
The maximum price at which the price charged by the contractor would indifferent to the cost to make the hoist is the
total differential cost or avoidable cost.
Direct materials 1,000
Direct labor 1,500
Variable overhead 500
Avoidable fixed overhead 1,200 x 0.30 360
Avoidable variable marketing cost 500 x 0.2 100
Maximum purchase price 3,460
101
. Answer: A
Direct materials 1,000
Direct labor 1,500
Variable overhead 500
Avoidable marketing costs 100
Opportunity cost [800 x (9,000 5,500 1,000)] 1,000 2,000
Maximum purchase price 5,100

A better understanding of the solution can be made by drawing a schedule to compute income for this alternative and
compare it with the income shown in solution for Question No. 97 as follows:

ModifiedRegularSales7,200,00022,200,000Variable production costs: In house production (2,000 x 3,000) 6,000,000


(800 x 5,500)4,400,000 Contractors cost 1,000 x 5,100 5,100,000Variable marketing costs Regular (2,000 x 500) +
(1,000 x 400) 1,400,000 Modified (800 x 1,000) 800,000Fixed costs. . 7,800,000Profit2,000,0001,900,000
Total profit (2,000,000 + 1,900,000) 3,900,000
102
. Answer: A
Direct material (6 lbs. P1.50) P9.00
Direct labor (0.25 hr. P7) 1.75
Direct machine cost (P10/blanket) 10.00
Variable overhead (0.25 hr. P3) 0.75
Administrative costs (P2,500/1,000) 2.50
Minimum bid price P24.00
103
. Answer: B
Using the full-cost criteria and the maximum allowable return specified, Marcus Fibers bid price per blanket would be:
Relevant costs (from Requirement 1) P24.00
Fixed overhead (0.25 hr. P8) 2.00
Subtotal P26.00
Allowable return (0.15* P26) 3.90
Bid price P29.90

*0.09/(1 0.40) = 0.15

You might also like